Elms Mock Board Reviewer

You might also like

Download as docx, pdf, or txt
Download as docx, pdf, or txt
You are on page 1of 121

CFAS

Question 1
It is an accounting method where the investment is initially recognized at cost and adjusted for
the post-acquisition change in the investor's share of the investee's net assets.
Response: Equity method
Feedback: Correct! The equity method is an accounting method whereby the investment is
initially recognized at cost and adjusted for the post-acquisition change in the investor's share
of the investee's net assets. With this method, the investor company reports the revenue
earned by the other company on its income statement, in an amount proportional to the
percentage of its equity investment in the other company.
Score: 1 out of 1 Yes
Question 2
What is the initial measurement of an internally generated intangible asset?
Response: Directly attributable cost incurred after the asset met the criteria for capitalizing
development cost
Feedback: Correct! An internally generated intangible asset is initially measured using directly
attributable cost incurred after the asset met the criteria for capitalizing development cost.
Score: 1 out of 1 Yes
Question 3
Which of the following is NOT an adjusting event after the reporting period?
Response: The settlement before the reporting period of a court case that confirms the entity
had a present value after the present obligation at the end of the reporting period.
Feedback: Correct! Because the settlement of a court case happened before the reporting
period, not after the reporting period.
Score: 1 out of 1 Yes
Question 4
What is the initial measurement of an intangible asset acquired through a government grant?
Response: Fair value or nominal account plus directly attributable expenditures
Feedback: Correct! Intangible asset acquired through a government grant is initially measured
at fair value or nominal account plus any directly attributable expenditures. Fair value
represents the estimated worth of various assets and liabilities that must be listed on a
company's books.
Score: 1 out of 1 Yes
Question 5
Shaina took the October 200B Certified Public Accountant Licensure Examination (CPALE)
and obtained a general average of 85%, with four (4) subjects higher than 75% and no grade
lower than 65% in the remaining two (2) subjects. Using the current Board of Accountancy
(BOA) Resolution, what is her status in the examination?
Response: Passed
Feedback: Correct! Shaina passed because she obtained a general average of 85%, with four
(4) subjects higher than 75% and no grade lower than 65% in the remaining two (2) subjects.
Score: 1 out of 1 Yes
Question 6
It is an amount by which the carrying amount of an asset exceeds its recoverable amount.
Response: Impairment loss
Feedback: Correct! Impairment loss exists when the carrying amount of an asset exceeds its
recoverable amount. When testing an asset for impairment, the total profit, cash flow, or other
benefit expected to be generated by that specific asset is periodically compared with its
current book value.
Score: 1 out of 1 Yes
Question 7
Which of the following DOES NOT result in a deferred tax asset?
Response: Taxable temporary differences
Feedback: Correct! This is the direct opposite of a deferred tax asset. The taxable temporary
difference will result in taxable amounts in a future period's taxable profit (or loss).
Score: 1 out of 1 Yes
Question 8
These events are indicative of conditions that arose after the reporting period.
Response: Non-adjusting events
Feedback: Correct! Non-adjusting events are indicative of conditions that arose after the
reporting period. Examples of which are declaration of dividends and initiation of litigation
against the company arising out after the reporting period.
Score: 1 out of 1 Yes
Question 9
Which condition must be met for an item to be recognized as an intangible asset other than
goodwill?
Response: The item is non-monetary, identifiably, and lacks physical substance.
Score: 1 out of 1 Yes
Question 10
Which of the following should NOT be included in the physical inventory of a company?
Response: Goods held on consignment from another company
Feedback: Correct! Goods held on consignment should not be included because another
company has a title (ownership) to the goods.
Score: 1 out of 1
FAR
Question 1
Adjustments for unpaid rendered services:
Response: Have an assets-and-revenues-account relationship
Feedback: Correct! Adjustments will have an assets-and-revenues-account relationship.
Score: 1 out of 1 Yes
Question 2
ML Company incurred the following costs in its production of Product Green: Beginning
finished goods of P10,000; Cost of goods manufactured amounting to P6,500 and ending
finished goods of P5,000. What amount will be reported as the cost of goods sold?
Response: P11,500
Feedback: Correct! Beg. Finished Inv. + COGM – End. Finished Inv. = COGS; P10,000 +
P6,500 - P5,000 = P11,5000
Score: 1 out of 1 Yes
Question 3
Renalie Company was incorporated on January 1, 2X21, with P5,000,000 from the issuance of
share capital and borrowed funds of P1,500,000. During the first year, the net income was
P2,500,000. On December 15, the entity paid a P500,000 cash dividend. On December 31,
2X21, the liabilities had increased to P1,800,000. What should be reported as total assets on
December 31, 2X21?
Response: P8,800,000
Feedback: Correct! Liabilities P1,800,000 + Share capital P5,000,000 + Retained earnings
P2,000,000 (P2,500,000 - P500,000) = P8,800,00
Score: 1 out of 1 Yes
Question 4
Yoshi Manufacturing incurred the following costs in its production of Product Y: Direct
Materials of P10,000; Direct Labor of P8,000; and Manufacturing Overhead of P7,500. How
much will Yoshi record as prime cost?
Response: P18,000
Feedback: Correct! DM + DL = Prime Cost; P10,000 + P8,000 = P18,000
Score: 1 out of 1 Yes
Question 5
X Company incurred the following costs in its production of Product A: Direct Materials of
P15,000; Direct Labor of P10,000; and Manufacturing Overhead of P12,000. Compute the cost
of goods manufactured if the company has P25,000 worth of beginning Work in Process (WIP)
inventory and P13,000 ending WIP inventory.
Response: P49,000
Feedback: Correct! Beg. WIP + (DM + DL + MO) – End. WIP = COGM; P25,000 +
(15,000+10,000+12,000) – P13,000 = P49,000
Score: 1 out of 1 Yes
Question 6
Adjustments for professional fees received but not yet rendered:
Response: Decrease liabilities and increase revenues
Feedback: Correct! Adjustments will consist of a debit (decrease) to unearned revenues (a
liability) and credit (increase) to a revenue account.
Score: 1 out of 1 Yes
Question 7
Given the following account balances of Jan Company for the month ended June 30,
2X22, how much is its total assets?

Revenues P85,000
Expenses:
Popcorn 22,800
Toppings and
seasonings 17,300
Employees’ wages
and benefits 10,700
Lease payments 24,000
Utilities 3,200
Advertising 900
Miscellaneous 300
Cash 6,200
Equipment 12,500
Accounts Payable 650
Wages Payable 1,200
Investment by Owner 12,500
Drawings by Owner 1,450
Response: P18,700
Feedback: Correct! Cash P6,200 + Equipment P12,500 = P18,700
Score: 1 out of 1 Yes

Question 8
A company has purchased a tract of land and plans to build a production plant on the land in
approximately five (5) years. The land will be idle during the five (5) years before construction.
The land should be reported as:
Response: Long-term investment
Feedback: Correct! Long-term investments include long-term assets such as land that a
company is not currently using in its operating activities.
Score: 1 out of 1 Yes
Question 9
The trial balance shows Prepaid Rent P1,575 and Rent Expense P0. If P800 is the remaining
balance of Prepaid Rent at the end of the period, the adjusting entry is:
Response: Dr. Rent Expense P775; Cr. Prepaid Rent P775
Feedback: Correct! Debiting Rent Expense for P775 and crediting Prepaid Rent for P775
(P1,575 – P800) will decrease Prepaid Rent and increase Rent Expense.
Score: 1 out of 1 Yes
Question 10
Given the following account balances of Jan Company for the month ended June 30,
2X18, how much is its net assets?

Revenues P85,000
Expenses:
Popcorn 22,800
Toppings and
seasonings 17,300
Employees’ wages
and benefits 10,700
Lease payments 24,000
Utilities 3,200
Advertising 900
Miscellaneous 300
Cash 6,200
Equipment 12,500
Accounts Payable 650
Wages Payable 1,200
Investment by Owner 12,500
Drawings by Owner 1,450
Response: P16,850
Feedback: Correct! Total assets P18,700 – Total liabilities P1,850 = P16,850

IA1
Question 1
Jade Company acquired two (2) items of machinery as follows:
 On December 31, 2X14, Jade Company purchased a machine in exchange for a noninterest-
bearing note requiring 10 payments of P500,000. The first payment was made on December
31, 2X15, and the others are due annually on December 31. The prevailing interest rate for this
type of note at the date of issuance was 12%. The present value of an ordinary annuity of 1 at
12% is 5.33 for nine (9) periods and 5.65 for 10 periods.
 On December 31, 2X14, Jade Company acquired used machinery by issuing the seller a two-
year, noninterest-bearing note for P3,000,000. The entity has paid a 12% interest for this type
of note in recent borrowing. The present value of 1 at 12% for two (2) years is .80, and the
present value of an ordinary annuity of 1 at 12% for two (2) years is 1.69.
What is the total cost of machinery?
Response: P5,225,000
Feedback:
Correct!

Present value of first note payable


P2,825,000
(500,000 x 5.65)
Present value of second note
2,400,000
payable (3,000,000 x .80)
January 25 P5,225,000
Score: 1 out of 1 Yes
Question 2
The following unadjusted cash balances are available for Joon Company for the month ended
November 30, 201A.
Cash balance per bank statement, November 30, 201A P 62, 305.75
Cash balance per company records, November 30, 201A
62,189.70
The bank statement disclosed the following information:
 Charges by the bank included a returned customer’s check for P690.70 because of insufficient
funds (NSF) and a service charge of P75 for November.
 Credits by the bank included a customer’s note for P6,000 plus interest of P60 that was
collected on November 29, 201A.
A review of the company records disclosed the following information:
 A deposit for P5,714.35 made on November 29, 201A did not appear on the bank statement.
 Customers’ checks totaling P1,637 were still on hand on November 30, 201A awaiting deposit.
 The following company checks were still outstanding as of November 30, 201A:
Check #145243 P480.95
Check #145247 735.90
Check #145250 1,316.25
Check #145257 for P456 in payment of a creditor account and included with the canceled
checks in the bank statement has been erroneously recorded in the company records as P96.
Using bank reconciliation, what is the unadjusted cash balance per bank?
Response: P62,305.75
Feedback: Correct! Per the given, this is the unadjusted cash balance.
Score: 1 out of 1 Yes
Question 3
YM Company conducted a physical count on December 31, 20X1, which revealed the total cost
of P3,600,000. However, the following items were excluded from the count:
 Goods sold to a customer, which are being held by the company on behalf of the customer.
The customer may call the company to deliver the goods anytime. The cost of the goods is
P200,000.
 A packing case containing a product costing P80,000 was standing in the shipping room when
the physical inventory was taken. It was not included in the inventory because it was marked
“hold for shipping instructions.”
 Goods in process costing P300,000 are held by an outside processor for further processing.
 Goods worth P50,000 shipped by a vendor FOB seller on December 28, 20X1, and received by
YM Company on January 10, 20X2.
What is the inventory on December 31, 20X1?
Response: P4,030,000
Feedback:
Correct!

Inventory per physical count P3,600,000


Inventory marked “hold for shipping
80,000
instructions”
Goods in process inventory 300,000
Goods shipped FOB seller or FOB shipping
50,000
point
Correct Inventory P4,030,000
Score: 1 out of 1 Yes
Question 4
On December 31, 20X1, Mashiho Corporation had accounts receivable of P750,000. On
January 1, 20X1, Allowance for Doubtful Accounts had a credit balance of P18,000. In 2020,
P30,000 of uncollectible accounts receivable were written off. Experience indicates that 3% of
accounts receivable became uncollectible. What should be the Bad Debt Expense for 20X1?
Response: P34,500
Feedback: Correct: The accounts written off during the year will result in a debit balance in
Allowance for Doubtful Accounts of P12,000 (P30,000 − P18,000) at the end of the year. As
indicated, 3% of accounts receivable are uncollectible, or P22,500 (P750,000 × 3%). Given a
debit balance of P12,000 in Allowance for Doubtful Accounts at the end of the year, the
adjusting entry at the end of the year is a debit to Bad Debt Expense of P34,50
0 (P22,500 + P12,000) and a credit to Allowance for Doubtful Accounts of P34,500.
Score: 1 out of 1 Yes
Question 5
Able Towing Company purchased a tow truck for P60,000 on January 1, 2018. It was originally
depreciated on a straight-line basis over 10 years with an assumed salvage value of P12,000.
On December 31, 2020, before adjusting entries had been made, the company decided to
change the remaining estimated life to four (4) years (including 2020) and the salvage value to
P2,000. What is the depreciation expense for 2020?
Response: P12,100
Feedback: Correct! First, calculate accumulated depreciation from January 1, 2018, through
December 31, 2019, which is P9,600 {[(P60,000 − P12,000)/10 years] × 2 years}. Next,
calculate the revised depreciable cost, which is P48,400 P60,000 − P9,600 − P2,000). Thus,
the depreciation expense for 2020 is P12,100 (P48,400/4).
Score: 1 out of 1 Yes
Question 6
DY Company provided the following information relating to accounts receivable for 2X14:

Accounts receivable on January 1 P1,300,000


Credit sales 5,400,000
Collections from customer, excluding
4,750,000
recovery
Accounts written off 125,000
Collection of accounts written off in a
prior year (customer credit was not 25,000
reestablished)
Estimated uncollectible receivables
per aging of receivables on December 165,000
31
On December 31, 2X14, what is the accounts receivable balance before Allowance for Doubtful
Accounts?
Response: P1,825,000
Feedback:
Correct! The accounts receivable balance is P1,825,000.

Accounts receivable – January 1 P1,300,000


Add: Credit sales 5,400,000
Total 6,700,000
Less: Collection from customers (P4,750,000 +
4,875,000
125,000)
Accounts receivable – December 31 P1,825,000
Score: 1 out of 1 Yes
Question 7
The following unadjusted cash balances are available for JK Company for the month ended
November 30, 201A.
Cash balance per bank statement, November 30, 201A P62,
305.75
Cash balance per company records, November 30, 201A
P62,189.70
The bank statement disclosed the following information:
 Charges by the bank included a returned customer’s check for P690.70 because of insufficient
funds (NSF) and a service charge of P75 for November.
 Credits by the bank included a customer’s note for P6,000 plus interest of P60 collected on
November 29, 201A.
A review of the company records disclosed the following information:
 A deposit for P5,714.35 made on November 29, 201A did not appear on the bank statement.
 Customers’ checks totaling P1,637 were still on hand on November 30, 201A awaiting deposit.
 The following company checks were still outstanding as of November 30, 201A:
Check #145243 P480.95
Check #145247 735.90
Check #145250 1,316.25
Check #145257 for P456 in payment of a creditor account and included with the canceled
checks in the bank statement has been erroneously recorded in the company records as P96.

Using bank reconciliation, what is the adjusted cash balance per book?
Response: P67,124
Feedback:
Correct! This is the adjusted cash balance per book.

Balance per book P62,189.70


Add: Notes collected by
P 6,000.00
the bank
Interest on notes collected 60.00 6,060.00
Total P68,249.70
Deduct:
Bank service charge P 75.00
Customer’s NSF check
690.70
returned
Check #145257 for P456
was erroneously recorded 360.00 1,125.70
as P96
Adjusted cash balance P 67,124.00
Score: 1 out of 1 Yes
Question 8
Boyoung Company ventured into the construction of a condominium in Pasig. The entity's
board of directors decided to hold this property to earn rentals by letting out space to business
executives in the area. The condominium construction was completed and placed in service on
January 1, 2X14. The construction cost was P50,000,000, with a useful life of 25 years and a
residual value of P5,000,000. An independent valuation expert provided the following fair value
at each subsequent year-end: December 31, 2X14 - P55,000,000; December 31, 2X15 -
P53,000,000; and December 31, 2X16 - P60,000,000. Under the fair value model, what amount
should be recognized as gain or loss from change in fair value for 2X14?
Response: P5,000,000 gain
Feedback: Correct! Fair Value on December 31, 2X14 P55,000,000 - Cost on January 1, 2X14
P50,000,000 = Gain P5,000,000
Score: 1 out of 1 Yes
Question 9
In January of the current year, Hope Company, which maintains a perpetual inventory system,
recorded the following information of its inventory:

Unit Units
Units Total cost on
cost hand
Balance -1/1 10,000 100 1,000,000 10,000
Purchased -1/7 6,000 300 1,800,000 16,000
Sold - 1/20 9,000 7,000
Purchased –
4,000 500 2,000,000 11,000
1/25
Using the moving average method, what should the company report as inventory on January
31?
Response: P3,225,000
Feedback:
Correct!

U
Uni nit Total
ts co cost
st
P
10, P1,00
January 1 10
000 0,000
0
6,0 30 1,800,
January 7
00 0 000
Balance
16, 17 2,800,
(2,800,000/
000 5 000
16,000)
January 20 (9,0 17 (1,575
- Sale 00) 5 ,000)
7,0 17 1,225,
Balance
00 5 000
4,0 50 2,000,
January 25
00 0 000
11, 29 P3,22
Balance
000 3 5,000
Score: 1 out of 1 Yes
Question 10
Hype Company acquired a tract of land containing an extractable natural resource. The entity
is required by the purchase contract to restore the land to a condition suitable for recreational
use after it has extracted the natural resources. A geological survey indicated that the
recoverable reserves will be 2,500,000 tons and that the extraction will be completed in five (5)
years. Relevant cost information shows the following: Land - P9,000,000; Exploration and
development cost - P1,000,000; Expected cash flow for restoration cost - P1,500,000; Credit-
adjusted risk-free interest rate - 10%; and PV of 1 at 10% for five (5) periods - 0.62. What is the
depletion charge per ton?
Response: P4.37
Feedback: Correct! Depletable Amount = Land Cost P9,000,000 + Exploration and
Development Cost P1,000,000 + Present Value of Expected Restoration Cost (P1,500,000 x
0.62) P930,000 = P10,930,000/2,500,000 = P4.37

IA2
Question 1
At the beginning of the current year, Ashe Company entered into a 10-year noncancelable
lease requiring year-end payments of P1,000,000. Ashe’s incremental borrowing rate is 12%,
while lessor’s implicit interest rate known to Ashe is 10%. Present value factors for an ordinary
annuity for 10 periods are 6.145 at 10% and 5.650 at 12%. On the same date, Ashe paid the
initial direct cost of P200,000 to negotiate and secure the leasing arrangement. Ownership of
the property remains with the lessor at the expiration of the lease. There is no purchase option.
The leased property has an estimated economic life of 12 years. What is the annual
depreciation of the right of use asset?
Response: P634,500
Feedback: Correct! Depreciation of right of use asset (6,345,000/10 years = P634,500)
Score: 1 out of 1 Yes
Question 2
At the end of the current year, Lessee Company leased machinery with the following
information:

Annual rental payable at the end of each year P1,000,000


Residual value guarantee 500,000
Payment to the lessor to obtain a long-term
300,000
lease
Cost of dismantling and restoring the asset as
390,000
required by the contract at present value
Annual executory cost paid by lessee 50,000
Lease term 4 years
Useful life of machinery 8 years
Implicit interest rate 10%
Present value of an ordinary annuity of 1 at
3.17
10% for four (4) periods
Present value of 1 at 10% for four (4) periods 0.68
Based on the given information, what is the lease liability at year-end?
Response: P2,861,000
Feedback: Correct! The lease liability at year-end is P2,861,000. Annual rental P1,000,000 –
Applicable to interest (10% x 3,510,000) P351,000 = Applicable to principle P649,000; Lease
liability P3,520,000 – Principal payment P649,000 = Lease Liability - December 31
= P2,861,000
Score: 1 out of 1 Yes
Question 3
These are components of defined benefit cost, EXCEPT:
Response: Contribution to the plan
Feedback: Correct! Contribution to the plan is not a component of defined benefit cost.
Score: 1 out of 1 Yes
Question 4
On November 1, 2X19, ABC Company discounted its note of P1,000,000 at 12% for one (1)
year. What is the net cash proceeds?
Response: P880,000
Feedback: Correct! Notes payable P1,000,000 – Discount P120,000 = Net cash
proceeds P880,000
Score: 1 out of 1 Yes
Question 5
Which of the following statements is INCORRECT about recognizing and measuring a defined
contribution plan?
Response: An entity shall not disclose the amount recognized as an expense for a defined
contribution plan.
Feedback: Correct! An entity shall disclose the amount recognized as an expense for a defined
contribution plan.
Score: 1 out of 1 Yes
Question 6
Juan Company includes one (1) coupon in each box of corn flakes it sells. A bowl is offered as
a premium to customers who send in 10 coupons and remittance of P10. The distribution cost
of the premium is P5. Experience indicates that only 30% of the coupons will be redeemed.
What is the premium liability on December 31, 2X19?
2X18 2X19
Boxes of corn flakes sold 2,000,000 2,500,000
Number of bowls
50,000 80,000
purchased at P50 each
Coupons redeemed 400,000 700,000
Response: P1,125,000
Feedback:
Correct!

Coupons expected to be
1,350,000
redeemed ( 4,500,000 x 30%)
Coupons redeemed 1,100,000
Balance 250,000
No. of coupons required for one
10
(1) premium
Expected premium to be
25,000
redeemed
Cost per premium 45
Estimated premium liability 1,125,000
Score: 1 out of 1 Yes
Question 7
Before deducting bonus and income taxes, TMI Company has a profit of P1,000,000. The
bonus rate and income tax rates are 10% and 30%, respectively. What is the bonus amount if it
is based on profit after bonus but before taxes?
Response: P90,909
Feedback:
Correct! P90,909 is the bonus based on profit after bonus but before taxes.
B = .10 x (P1,000,000 - B)
B = P100,000 - .10B
B = P100,000/1.10
B = P90,909
Score: 1 out of 1 Yes
Question 8
At the beginning of the current year, Ashe Company entered into a 10-year noncancelable
lease requiring year-end payments of P1,000,000. Ashe’s incremental borrowing rate is 12%,
while lessor’s implicit interest rate known to Ashe is 10%. Present value factors for an ordinary
annuity for 10 periods are 6.145 at 10% and 5.650 at 12%. On the same date, Ashe paid the
initial direct cost of P200,000 to negotiate and secure the leasing arrangement. Ownership of
the property remains with the lessor at the expiration of the lease. There is no purchase option.
The leased property has an estimated economic life of 12 years. What amount should be
capitalized initially as the cost of the right of use asset?
Response: P6,345,000
Feedback: Correct! Present value of rentals (1,000,000 X 6.145) P6,145,000 + Initial direct cost
P200,000 = Total cost of property P6,345,000.
Score: 1 out of 1 Yes
Question 9
On December 31, 2X19, and 2X18, Mashiho Company had 100,000 ordinary shares and
10,000 cumulative preference shares of 5%, P100 par value. No dividends were declared on
either the preference or ordinary shares in 2X19 or 2X18. Net income for the current year was
P900,000. What amount should be reported as basic earnings per share?
Response: P8.50
Feedback: Correct! Net income P900,000 – Preference dividend (P1,000,000 x 5%) P50,000 =
Net income to ordinary shares P850,000 / 100,000 ordinary shares = P8.50
Score: 1 out of 1 Yes
Question 10
Post-employment employee benefits include all of the following, EXCEPT:
Response: Long-term disability benefits
Feedback: Correct! Long-term disability benefits are under other long-term employee benefits.

IA3
Question 1
Jack Company (JC) earns a net profit of P100,000 and has 5,000,000 common shares
outstanding that sell on the open market for an average of P20 per share. Also, 500,000
options are outstanding that can be converted to JC’s common stock at P10 each. Which of the
following is the diluted earnings per share (EPS) of JC Company?
Response: 0.0196
Feedback: Correct! Net Profit / Diluted shares; P100,000 / 5,114,286 = 0.0916
Score: 1 out of 1 Yes
Question 2
Assume that ABC Company had post-tax profits for 2X21 of P1,375,000 and issued share
capital of P1,650,000 comprising 1,000,000 ordinary shares of P0.65 each and 1,000,0000 at
P1, 10% preference shares classified as equity. Which of the following are the basic earnings
per share?
Response: 1.28
Feedback: Correct! Profit attributable to ordinary shares / Total preference shares = Basic
earnings per share; P1,275,000/1,000,000 shares = 1.28
Score: 1 out of 1 Yes
Question 3
Juan Company provided the following data on December 31, 20x1:
 Cash, P27,500
 Accounts receivable, P583,000
 Prepayments, P66,000
 Inventories, P93,000
 Investment in associate, P131,600
 Property, plant, & equipment, P3,575,000
 Dep. and impairment, P770,000
 Software net of amortization and impairment, P11,000
 Deferred tax asset, P8,500
 Bank overdraft, P88,000
 Trade payables, P500,000
 Interest payable, P2,200
 Current tax liability, P297,000
 Provision for warranty, P4,400
 Employee benefit obligation,
 Current, P6,000
 Non-current, P5,000
 Finance lease liability,
 Current, P24,400
 Non-current, 34,600
 Bank Loan due in 20x3, P55,0000
 Share capital, P33,000
 Retained earnings, P2,673,000
Which of the following is the total non-current assets?
Response: P2,956,100
Feedback: Correct! Non-current assets are the investment in an associate, property, plant, and
equipment, software, and deferred tax asset.
Score: 1 out of 1 Yes
Question 4
ABC Company earns a net profit of P100,000 and has 5,000,000 common shares outstanding
that sell on the open market for an average of P20 per share. Also, 500,000 options are
outstanding that can be converted to ABC’s common stock at P10 each. Which of the following
is the number of diluted shares?
Response: 5,250,000 shares
Feedback: Correct! Options x average exercise price / average market price; (500,000 x
10)/20= 250,000; options - shares that could have been purchased with the proceeds from the
options; 500,000 – 250,000 = 250,000 shares plus common shares of 5,000,000 = 5,250,000.
Score: 1 out of 1 Yes
Question 5
The following are the accounts presented on Jeon Company’s Statement of Financial Position:

Cash 337,770
Accounts receivable 87,480
Inventories 374,625
Building & equipment 3,847,500
Acc. Depreciation 1,265,625
Accounts payable 202,500
Salaries payable 22,275
Total liabilities 224,775

The cash book shows the receipts and payments during 2X21 as follows:
Total liabilities
Cash Receipts
Collections on AR 719,685
Cash sales 923,400

Cash Payments
AP for merchandise 1,100,385
Salaries 216,675
Other operating expenses 60,750
Withdrawals by the owner 121,500

Supplementary information:
Sales returns and allowances 36,450
Cash discounts to customers 12,150
AR written off as worthless 6,075
Cash discounts on purchases 22,275
Purchase returns & allowances 19,642.50

Balances were taken on December 31, 2X21 from the supplementary analysis:
Accounts Receivable 153,900
Accounts Payable 172,125

How much is the amount of credit sales?


Response: P840,780
Feedback: Correct! Collections on accounts receivable + Accounts Receivable (Dec. 31) +
Accounts receivable written off as worthless + Sales returns and allowances + Cash discounts
taken by the customers - Accounts receivable = 719,685.00 + 153,900.00 + 6,075.00 +
36,450.00 + 12,150.00 - 87,480.00 = 840,780.00
Score: 1 out of 1 Yes
Question 6
Assume that ABC Corp. purchased two (2) P0.65 shares at a market price of P5 each in XYZ
Company on January 01, 20x1 and that on January 02, 20x1, the company offered a 1:2 rights
issue at P4.25 per share. If ABC Corp. had bought the shares on January 01, 20x1, how much
is the price of the two (2) shares?
Response: 10
Feedback: Correct! Two (2) shares at the market price of P5 each on January 01, 2ax1 equals
10, 2 x P5 each share.
Score: 1 out of 1 Yes
Question 7
Juan Company provided the following data on December 31, 20x1:
 Cash, P27,500
 Accounts receivable, P583,000
 Prepayments, P66,000
 Inventories, P93,000
 Investment in associate, P131,600
 Property, plant, & equipment, P3,575,000
 Dep. and impairment, P770,000
 Software net of amortization and impairment, P11,000
 Deferred tax asset, P8,500
 Bank overdraft, P88,000
 Trade payables, P500,000
 Interest payable, P2,200
 Current tax liability, P297,000
 Provision for warranty, P4,400
 Employee benefit obligation,
 Current, P6,000
 Non-current, P5,000
 Finance lease liability,
 Current, P24,400
 Non-current, 34,600
 Bank Loan due in 20x3, P55,0000
 Share capital, P33,000
 Retained earnings, P2,673,000
Which of the following are the total current liabilities?
Response: P925,000
Feedback: Correct! Bank overdraft + Trade payables + Interest payable + Current tax liability +
Provision for warranty + Employee benefit obligation, current portion P6,000 + Finance lease
liability, current portion P24,400; 88,000 + 500,000 + 2,200 + 297,000 + 4,400 + 6,000 +
24,400 = 925,000
Score: 1 out of 1 Yes
Question 8
Assume that Hero Company had post-tax profits for 2X21 of P1,375,000 and issued share
capital of P1,650,000 comprising 1,000,000 ordinary shares of P0.65 each and 1,000,0000 at
P1, 10% preference shares classified as equity. Which of the following are the dividends on
preference shares classified as equity?
Response: P100,000
Feedback: Correct! 1,000,000 at P1, 10% preference shares classified as equity
Score: 1 out of 1 Yes
Question 9
ABC Company’s income statement reports a depreciation expense of P2,500,000 and net
income of P23,800,000 for the year. The company also sold two (2) major equipment: EQ001
which costs P10,000,000 and sold at P10,500,000, and EQ002 which costs P12,800,000, and
sold at P11,700,000. If there were no other non-cash items, which amount is the net cash
provided by the operating activities?
Response: P26,900,000
Feedback: Correct! Net income +/- adjustments: Depre. Exp. + gain on disposal – loss on
disposal = Net cash provided by operating activities; P23,800,000 + P2,500,000 – P500,000 +
P1,100,000 = P26,900,000
Score: 1 out of 1 Yes
Question 10
The following are the accounts presented on Jeon Company’s Statement of Financial Position:

Cash 337,770
Accounts receivable 87,480
Inventories 374,625
Building & equipment 3,847,500
Acc. Depreciation 1,265,625
Accounts payable 202,500
Salaries payable 22,275
Total liabilities 224,775

The cash book shows the receipts and payments during 2X21 as follows:
Total liabilities
Cash Receipts
Collections on AR 719,685
Cash sales 923,400

Cash Payments
AP for merchandise 1,100,385
Salaries 216,675
Other operating expenses 60,750
Withdrawals by the owner 121,500

Supplementary information:
Sales returns and allowances 36,450
Cash discounts to customers 12,150
AR written off as worthless 6,075
Cash discounts on purchases 22,275
Purchase returns & allowances 19,642.50

Balances were taken on December 31, 2X21 from the supplementary analysis:
Accounts Receivable 153,900
Accounts Payable 172,125

How much is the amount of purchases?


Feedback: Correct! Accounts payable for merchandise + (Accounts Payable (Dec. 31) + Cash
discounts taken on purchases + Purchase returns and allowances - Accounts payable) =
P1,100,385.00 + (172,125.00 + 22,275.00 + 19,642.50 - 202,500.00) = P1,111,927.50

AUDITING THEORY
Question 1
In the first audit of a client, an auditor was not able to gather sufficient evidence about the
consistent application of accounting principles between the current and prior year, as well as
the amounts of assets or liabilities at the beginning of the current year. This was because of the
client’s record retention policies. If the amounts in question could materially affect current
operating results, the auditor would:
Response: Be unable to express an opinion on the current year’s results of operations and
cash flows
Feedback: CORRECT! The scope limitation will affect the year’s beginning balances and affect
the current year’s operations and cash flows. Therefore, the auditor will disclaim an opinion.
Correct answer: Be unable to express an opinion on the current year’s results of operations
and cash flows
Score: 1 out of 1 Yes
Question 2
Which of the following is an audit procedure that an auditor would MOST likely perform
concerning litigation, claims, and assessments?
Response: Discuss its policies and procedures adopted for evaluating and accounting for
litigation, claims, and assessments with management.
Feedback: CORRECT! Auditors must discuss its policies and procedures for evaluating and
accounting for litigation, claims, and assessments with management.
Correct answer: Discuss its policies and procedures adopted for evaluating and accounting for
litigation, claims, and assessments with management.
Score: 1 out of 1 Yes
Question 3
An auditor who discovers that a client’s employees paid small bribes to municipal officials
would MOST likely withdraw from the engagement if:
Response: Management fails to take the appropriate remedial action.
Feedback: CORRECT! Management’s failure to take the appropriate remedial action is
particularly problematic since it may affect the auditor’s ability to rely on management
representation and may therefore lead to withdrawal.
Correct answer: Management fails to take the appropriate remedial action.
Score: 1 out of 1 Yes
Question 4
These provide additional evidence about conditions that existed at the balance sheet date and
affected the estimates part of the financial statement preparation process.
Response: Adjusting events
Feedback: CORRECT! They are also known as Type I events and may require adjusting
amounts in the financial statements.
Correct answer: Adjusting events
Score: 1 out of 1 Yes
Question 5
Confirmation is most likely to be a relevant form of evidence about assertions on accounts
receivable when the auditor has concerns about the receivables’:
Response: Existence
Feedback: CORRECT! A confirmation addresses whether the entity replying to the confirmation
believes that a debt exists.
Correct answer: Existence
Score: 1 out of 1 Yes
Question 6
It is the risk that a misstatement could occur in an assertion about a class of transaction,
account balance, or disclosure and that could be material, either individually or when
aggregated with other misstatements, which will not be prevented, or detected and corrected,
on a timely basis by the entity’s internal control.
Response: Control risk
Feedback: CORRECT! Control risk is related to the effectiveness of the client’s internal control.
It also exists independently in the audit of financial statements and is assessed using the
auditor’s judgment.
Correct answer: Control risk
Score: 1 out of 1 Yes
Question 7
Which of the following procedures would an auditor MOST likely perform in obtaining evidence
about subsequent events?
Response: Investigate changes in long-term debt occurring after year-end.
Feedback: CORRECT! The auditor obtains evidence about subsequent events to determine
unusual events that may happen. Changes in long-term debt occurring after year-end may
require disclosure in the notes. The changes need to be disclosed because they are material,
and the stockholders need to be aware of them.
Correct answer: Investigate changes in long-term debt occurring after year-end.
Score: 1 out of 1 Yes
Question 8
Which of the following procedures should an auditor generally perform regarding subsequent
events?
Response: Compare the latest available interim financial statements with the financial
statements being audited.
Feedback: CORRECT! The professional standards state that, generally, the auditor should
compare the latest available interim financial statements with the financial statements being
audited.
Correct answer: Compare the latest available interim financial statements with the financial
statements being audited.
Score: 1 out of 1 Yes
Question 9
As the acceptable level of detection risk decreases, an auditor may:
Response: Postpone the planned timing of substantive tests from interim dates to the year-end.
Feedback: CORRECT! Postponement of interim substantive tests to year-end decreases
detection risk by reducing the risk for the period after the performance of those tests; other
approaches to decreasing detection risk include changing to more effective substantive tests
and increasing their extent.
Correct answer: Postpone the planned timing of substantive tests from interim dates to the
year-end.
Score: 1 out of 1 Yes
Question 10
An auditor MAY NOT issue a qualified opinion when:
Response: The auditor lacks independence from the audited entity.
Feedback: CORRECT! An auditor who lacks independence must disclaim an opinion, not
qualify an opinion.
Correct answer: The auditor lacks independence from the audited entity.

AUDIT OF ASSET
Question 1
Using the information in Case No. 3, what is the increase in the accounts payable balance on
June 30, 20x2?
Response: P44,100
Feedback: CORRECT! The accounts payable balance as of June 30, 20x2 is as
follows: P27,900 + P16,200 = P44,100.
Score: 1 out of 1 Yes
Question 2
Using the information in Case No. 5, what is the accumulated depreciation – machinery and
equipment on December 31, 20x6?
Response: P8,556,875
Feedback:
CORRECT! To compute the accumulated depreciation, determine first the depreciation
expense recognized in the previous years and 20x6. The accumulated depreciation in 20x6 is
already given.

Machinery and equipment,


Jan. 1, 201F P22,500,000
Less: Machine stolen 575,000
Balance P21,925,000
P2,192,50
Depreciation rate 10%
0
Machine stolen (P575,000 x
14,375
10% x 3/12)
Machine purchased July 1
87,500
(P7,750,000 x 10% x 6/12)
P2,594,37
Depreciation expense in 20x6
5
Remember to depreciate the stolen machine and the newly acquired machine during 20x6
since they will also be depreciated.

Balance, Jan. 1, 20x6 P6,250,000


Depreciation for 20x6 2,594,375
Machine stolen (P575,000/10 x 5) (287,500)
Balance, Dec. 31, 20x6 P8,556,875
Score: 1 out of 1 Yes
Question 3
Using the information in Case No. 3, what is the correct inventory on June 30, 20x2?
Response: P144,000
Feedback:
CORRECT! The inventory balance as of June 30, 20x2 is computed as follows: P105,000 +
P27,900 +P11,100 = P144,000.
The two (2) items were included since they are still in transit as of June 30, 20x2. Since all
purchases are shipped to the FOB shipping point, the goods are already owned by Hamilton.
Score: 1 out of 1 Yes
Question 4
Using the information in Case No. 4, what amount of unrealized gain should be shown in the
20x5 statement of changes in equity?
Response: P10,982
Feedback:
CORRECT! This can be determined as follows:

Unrealized
Year Gain
(Loss)
20x2 P(5,893)
20x3 26,455
20x4 (1,222)
20x5 (8,358)
Cumulative
P10,982
Gain
Score: 1 out of 1 Yes
Question 5
Using the information in Case No. 2, what is the net realizable value (NRV) of Gibas
Corporation’s accounts receivable on December 31, 20x1?
Response: P3,970,800
Feedback:
CORRECT! This is computed by deducting the required allowance from the accounts
receivable balance.
Accounts receivable (P4,600,000 –
P4,480,000
P120,000)
Required allowance 509,200
Net realizable value P3,970,800

The P120,000 should be deducted because it was already written off.


Score: 1 out of 1 Yes
Question 6
Using the information in Case No. 4, what amount of unrealized gain should be shown as a
component of other comprehensive income in the 20x3 statement of comprehensive income?
Response: P26,455
Feedback:
CORRECT! This amount is computed as follows:

Fair value, December 20x3 P1,075,000


Carrying value, Dec. 20x3
Fair value, 20x2 P1,065,000
Premium amortization, 2019 (16,455) 1,048,545
Unrealized gain P26,455

An amortization table can also be used as a reference to determine the unrealized gain.
Score: 1 out of 1 Yes
Question 7
Using the information in Case No. 1, what is the amount of cash shortage chargeable against
the cashier?
Response: P168,510
Feedback:
CORRECT! The amount can be computed as follows:

Total Accountability 252,760


Cash in Bank, April, 15, 20X1 (84,250)
Cash shortage chargeable to the
P168,510
cashier
Score: 1 out of 1 Yes
Question 8
Using the information in Case No. 1, what is the cashier’s accountability (correct cash balance
before shortage) on April 15, 20x1?
Response: P252,760
Feedback:
CORRECT! The cash balance can be computed as follows:

Cash balance, December 31,


P32,500
20X0
Collections 5,654,070
Disbursements (5,433,810)
Cash balance, April 15, 20X1
P252,760
(Accountability)
Score: 1 out of 1 Yes
Question 9
Using the information in Case No. 5, what is the accumulated depreciation – delivery
equipment on December 31, 20x6?
Response: P4,800,000
Feedback:
CORRECT! To determine the accumulated depreciation, compute first the depreciation
expense recognized in 20x6.

Depreciation on Jan. 1, 20x6,


P900,000
balance
Less: Depreciation on truck traded
180,000 P720,000
in, Jan. 1, 20x6 (900,000 x 2/10*)
Depreciation on truck purchased
480,000
Jan. 2, 20x6 (P1,200,000 x 4/10*)
Depreciation expense in 20x6 P1,200,000
*SYD = 4 (4+1/2) = 10

Here is the computation of accumulated depreciation:

Balance, Jan. 1, 20x6 P4,230,000


Depreciation expense in 20x6 1,200,000
Truck traded in
(P900,000 cost – P270,000 book value) (630,000)
Balance, Dec. 31, 20x6 P4,800,000

Score: 1 out of 1 Yes


Question 10
Using the information in Case No. 2, what is the balance of the Allowance for Bad Debts
account on December 31, 20x1 (before year-end adjustment)?
Response: P326,000
Feedback:
CORRECT! This amount pertains to the balance of allowance before adjustment for the bad
debts expense during the year.

Allowance for bad debts, Jan. 1 P143,000


Add: Bad debts expense (P15,000,000 x 2%) 300,000
Recovery 143,000
Accounts written off (P140,000 + P120,000) (260,000)
Allowance for bad debts, Dec. 31 P326,000

AUDIT OF LIABILITIES AND CORRECTION OF ERRORS


Question 1
Using the information in Case No. 7, what amount of noncurrent liabilities would Treasure
Corporation report in its March 31, 20x3 statement of financial position?
Response: P16,164,940
Feedback:
CORRECT! This is the amount of noncurrent liabilities, including the bonds payable and the
noncurrent portion of notes payable. All other liabilities are classified as current.

Bonds Payable P10,764,940


Noncurrent portion of
notes payable
5,400,000
(P7,000,000 –
P1,600,000)
Total noncurrent liabilities P16,164,940

The carrying value of bonds payable is computed as follows:

Carrying value, December


P10,742,660
31, 20x2
Discount amortization,
Jan 1 – Mar 31, 20x3 22,280
(P44,560 x 3/6)
Carrying value, March 31,
P10,764,940
20x3

The amortization table is shown below:

Interes Interest Discou


Carrying
Date t Paid Expens nt
Amount
(5%) e (6%) Amort.
07/20x 10,623,55
1 2
12/20x 600,00 637,41 10,660,96
37,413
1 0 3 5
07/20x 600,00 639,65 10,700,62
39,658
2 0 8 3
12/20x 600,00 642,03 10,742,66
42,037
2 0 7 0
07/20x 600,00 644,56 10,787,22
44,560
3 0 0 0
12/20x 600,00 647,23 10,834,45
47,233
3 0 3 3
07/20x 600,00 650,06 10,884,52
50,067
4 0 7 0
Score: 1 out of 1 Yes
Question 2
Using the information in Case No. 6, what amount of noncurrent liabilities should be reported
on the December 31, 20x1, statement of financial position?
Response: P6,000,000
Feedback: CORRECT! This amount pertains to the noncurrent portion of the long-term debt
amounting to P6,750,000. As of December 31, 20x1, the P750,000 is already considered
current since its maturity date is less than a year as of the end of the reporting period.
Score: 1 out of 1 Yes
Question 3
Using the information in Case No. 8, what is the corrected net income for 20x3?
Response: P33,300
Feedback:
CORRECT! This is the corrected net income after all the adjustments were made.

20x2 20x3
Unadjusted
P39,000
balances P45,000
Inventory-
overstated
20x2 (42,000) 42,000
20x3 (48,000)
Inventory-
understated
20x1 (36,000)
Prepaid expenses
omitted
20x1 (5,400)

20x2 4,200
(4,200)
20x3 3,000
Deferred revenues
omitted

20x2 2,400
(2,400)
Accrued expenses
omitted
20x1 1,200

20x2
(450) 450
20x3 (600)
Accrued revenues
omitted

20x2 750
(750)
(P35,10
Adjusted balances P33,300
0)
Score: 1 out of 1 Yes
Question 4
Using the information in Case No. 8, what is Harlequin Company’s total assets as of
December 31, 20x4?
Response: P316,500
Feedback:
CORRECT! This is the adjusted balance of total assets.

Unadjusted balance P258,000


Adjustments:
Inventory 54,000
Prepaid Expenses 3,600
Accrued revenue 900
Adjusted balance of
P316,500
total assets
Score: 1 out of 1 Yes
Question 5
Using the information in Case No. 8, what is the corrected net income for 20x4?
Response: P135,000
Feedback:
CORRECT! This is the corrected net income after all the adjustments were made.
20x3 20x4
Unadjusted balances P39,000 P33,000
Inventory-overstated
20x2 42,000
20x3 (48,000) 48,000
Inventory-understated
20x4 54,000
Prepaid expenses omitted
20x2 (4,200)
20x3 3,000 (3,000)
20x4 3,600
Deferred revenues omitted
20x2 2,400
20x4 (1,800)
Accrued expenses omitted
20x2 450
20x3 (600) 600
20x4 (300)
Accrued revenues omitted
20x2 (750)
20x4 900
Adjusted balances P33,300 P135,000
Score: 1 out of 1 Yes
Question 6
Using the information in Case No. 8, what is the Retained Earnings balance on December 31,
20x4?
Response: P218,400
Feedback:
CORRECT! This is the adjusted balance of retained earnings. Only those errors that occurred
in 20x4 affect retained earnings as of December 31, 20x4. The errors in 20x4 have the same
effect on retained earnings and net income.

20x4 NI 20x4 RE
Unadjusted P162,000
balances P33,000
Inventory-
understated
20x4 54,000 54,000
Prepaid expenses
omitted
20x4 3,600 3,600
Deferred revenues
omitted
20x4 (1,800) (1,800)
Accrued expenses
omitted

20x4 (300)
(300)
Accrued revenues
omitted

20x4
900 900
Adjusted balances P135,000 P218,400
Score: 1 out of 1 Yes
Question 7
Using the information in Case No. 8, what is the corrected net loss for 20x2?
Response: P35,100
Feedback:
CORRECT! This is the corrected net loss after all the adjustments were made.

Net income 20x2


Unadjusted
balances P45,000
Inventory-
overstated
20x2 (42,000)
Inventory-
understated

20x1
(36,000)
Prepaid expenses
omitted
20x1
(5,400)
20x2 4,200
Deferred revenues
omitted

20x2
(2,400)
Accrued expenses
omitted
20x1 1,200

20x2
(450)
Accrued revenues
omitted
20x2 750

Adjusted balances
(P35,100)
Score: 1 out of 1 Yes
Question 8
Using the information in Case No. 6, what amount of current liabilities should be reported on
the December 31, 20x1 statement of financial position?
Response:
P7,300,000
Feedback:
CORRECT! The following are the items classified as current liabilities:

Item (a) Short term notes


P4,200,000
payable
Item (b) Short-term obligation 250,000
Item (c) Debt callable on
600,000
demand
Item (d) Current portion of long-
750,000
term debt
Item (e) Long-term obligation
1,500,000
with breach of covenant
Total current liabilities P7,300,000
Score: 1 out of 1 Yes
Question 9
Using the information in Case No. 7, how much was received by Luca from the sale of bonds
on July 1, 20x1? Round off the present value factor in five (5) decimal places.
Response: P10,623,552
Feedback:
CORRECT! The requirement is to compute the proceeds from the sale of bonds. This is
calculated by determining the present value of principal and interest payments.

PV of principal (P12,000,000 x 0.31180) P3,741,600


PV of interest payments (P600,000 x 11.46992) 6,881,952
Proceeds P10,623,552
Score: 1 out of 1 Yes
Question 10
Using the information in Case No. 7, what is the balance of estimated warranties payable on
March 31, 20x3?
Response: P375,000
Feedback:
CORRECT! This is the estimated warranties payable on March 31, 20x2.

Balance, April 1, 20x2 P300,000


Add: Warranty expense
640,000
for the current year
Total 940,000
Less: Actual warranty
565,000
costs
Estimated warranty
P375,000
outstanding
Score: 1 out of 1

AUDIT OF INCOME AND EQUITY


Question 1
Using the information in Case No. 10, what is the balance of retained earnings before an
appropriation for treasury shares on December 31, 20x2?
Response: P16,790,662
Feedback:
CORRECT! Continuing the computation in the previous item, compute the dividends declared
during 20x2:
Shares Issued and Cash Stock
Outstanding Dividends Dividend
Ordinar
Ordinar Preferen Preferenc
y
y ce e (6.25%)
(P0.50)
20x
2
Jan
330,000
1
Feb
. 1 25,500
Apr
. 28 30,000
Apr 385,50 P192,7
. 30 0 45,000 50 P112,500
Ma
y 3,855
154,200
19
Oct P192,7
385,500 48,855 P122,138
. 31 50

20x2
Balance, 20x1 P14,895,000
Apr. 30 Dividends (305,250)
May 19 Dividends (154,200)
Oct. 31 Dividends (314,888)
Dec. 31 Net income 2,670,000
Balance, Dec. 31 P16,790,662
Score: 1 out of 1 Yes
Question 2
Using the information in Case No. 10, what is the preference share capital on December 31,
20x2?
Response: P1,954,200
Feedback:
INCORRECT! Take note of the transactions involving ordinary share capital.

Preference
Share
20x1
Jan. 1 P2,400,000
Sept. 30 Conversion 600,000
Balance, Dec. 31 P1,800,000

20x2
May 16. Cash Dividend 154,200
Balance, Dec. 31 P1,954,200
Score: 1 out of 1 Yes
Question 3
Using the information in Case No. 10, what is the balance of retained earnings before an
appropriation for treasury shares on December 31, 20x1?
Response: P14,895,000
Feedback:
CORRECT! To compute the retained earnings balance, determine the value of dividends
declared to be deducted from the beginning balance of retained earnings.

Shares Issued and Cash Stock


Outstanding Dividends Dividend
Ordinary Preference
Ordinary Preference
(P0.50) (6.25%)
20x1
Jan. 1 360,000 60,000
Feb. 1 (45,000)
Apr. 30 315,000 60,000 P157,500 P150,000
Aug. 30 15,000 (15,000)
Oct. 31 330,000 45,000 P165,000 P112,500

RE
20x1
Jan. 1 P13,500,000
Apr. 30 Dividends (307,500)
Oct. 31 Dividends (277,500)
Dec. 31 Net income 1,980,000
Balance, Dec. 31 P14,895,000
Score: 1 out of 1 Yes
Question 4
Using the information in Case No. 9, what is the compensation expense to be recognized in
year 1
Response: P225,000
Feedback:
CORRECT! The compensation expense in 20x1 is computed as follows:
P10 x 450 employees x 100 shares x ½ = P225,000
By the end of year 1, the entity expects that the shares will vest at the end of year 2.
Score: 1 out of 1 Yes
Question 5
Using the information in Case No. 9, what is the amount of compensation expense to be
recognized in year 2?
Response: P57,000
Feedback:
CORRECT! This is the compensation expense in year 2.
(P10 x 423 x 100 x 2/3) – P225,000 = P57,000
Score: 1 out of 1 Yes
Question 6
Using the information in Case No. 10, what is the share premium on December 31, 20x2?
Response: P10,838,100
Feedback:
CORRECT! The following are the transactions involving share premium:

SP
20x1
Jan. 1 P9,000,000
Sep. 30 525,000
Dec. 21 840,000
Balances, Dec. 31 P10,365,000
20x2
Feb. 1 (36,900)
Apr. 15 510,000
Balances, Dec. 31 P10,838,100
Score: 1 out of 1 Yes
Question 7
Using the information in Case No. 9, what amount should the entity report as share options
outstanding at the end of year 3?
Response: P428,000
Feedback:
CORRECT! The share options outstanding are equal to the cumulative compensation expense
for the period.

P10 x 428 x 100 = P428,000


Score: 1 out of 1 Yes
Question 8
Using the information in Case No. 10, what is the share premium on December 31, 20x1?
Response: P10,365,000
Feedback:
CORRECT! The following are the transactions involving share premium during 20x1:

Share premium
20x1
Jan. 1 P9,000,000
Sep. 30 Conversion 525,000
Dec. 21 Subscription 840,000
Balance, Dec. 31 P10,365,000
Score: 1 out of 1 Yes
Question 9
Using the information in Case No. 9, what is the amount of compensation expense to be
recognized in year 3?
Response: P146,000
Feedback:
CORRECT! This is the compensation expense in year 3.

(P10 x 428 x 100 x 3/3) – P282,000 = P146,000


Score: 1 out of 1 Yes
Question 10
Using the information in Case No. 9, what amount should the entity report as share options
outstanding at the end of year 2?
Response: P282,000
Feedback: CORRECT! The share options outstanding are equal to the cumulative
compensation expense for the period: P10 x 423 x 100 x 2/3 = P282,000.

AUDIT AND ASSURANCE IN SPECIALIZED INDUSTRIES


Question 1
In auditing the expected credit loss (ECL) set by a proprietary educational institution, the
auditor may conduct the following, EXCEPT:
Response: Verify whether the company followed the Five-Step Model provided by PFRS 15.
Feedback: CORRECT! This is one of the procedures in auditing the revenue of a proprietary
educational institution.
Score: 1 out of 1 Yes
Question 2
All of the following are procedures that may be conducted by an auditor upon the audit of a
mining company’s inventory, EXCEPT:
Response: Perform a test on the fixed-asset roll forward of the company.
Feedback: CORRECT! This is an audit procedure in auditing the mining company’s property,
plant, and equipment (PPE).
Score: 1 out of 1 Yes
Question 3
All of the following are substantive procedures that an auditor may perform in auditing the
underwriting process of insurance companies, EXCEPT:
Response: Agree the sub-ledger of premiums ceded and assumed to the general ledger
control accounts.
Feedback: CORRECT! This substantive procedure can be performed in auditing reinsurance.
Score: 1 out of 1 Yes
Question 4
In an audit of the lending process, which of the following is the MOST possible auditor’s
response to the risk that all the outstanding loan balances do not exist?
Response: The auditor may confirm balances with the bank's customers.
Feedback: CORRECT! The auditor may also inspect and review loan documents such as
promissory notes.
Score: 1 out of 1 Yes
Question 5
Which of the following is NOT a step in the Five-Step Model of recognizing revenue under the
Philippine Financial Reporting Standard (PFRS) 15?
Response: Understand the company’s technology roadmap plan and strategy related to asset
replacement.
Feedback: CORRECT! This procedure is conducted in auditing the property and equipment of
telecommunication companies.
Score: 1 out of 1 Yes
Question 6
All of the following are procedures that may be conducted by an auditor upon the audit of a
real estate company’s receivable, EXCEPT:
Response: Trace the accumulated costs to supporting documents such as invoices and
accomplishment reports from the contractors and official receipts
Feedback: CORRECT! This audit procedure is for auditing a real estate’s revenue recognition
process.
Score: 1 out of 1 Yes
Question 7
All of the following are substantive procedures that an auditor may perform in auditing the
contributions received by not-for-profit entities (NPOs), EXCEPT:
Response: Send confirmations to the security dealers to assess its fair value.
Feedback: CORRECT! This audit procedure is done in auditing the NPO’s investment.
Score: 1 out of 1 Yes
Question 8
Which of the following Philippine Standard Auditing (PSA) the auditor must observe upon the
use of an auditor’s expert to obtain audit evidence?
Response: PSA 620
Feedback: CORRECT! It is likely that in an audit of entities in the specialized industries,
auditors use an expert to obtain audit evidence. It is because despite being competent to
perform the engagement, the audit firm may not have the necessary specific expertise in some
areas. In such a case, the auditors must follow the requirements and principles set in PSA 620
– Using the Work of an Auditor’s Expert.
Score: 1 out of 1 Yes
Question 9
All of the following are characteristics of telecommunication companies that an auditor must
consider (key audit considerations), EXCEPT:
Response: They are usually private entities, and thus, their financial reporting is not that
stringent.
Feedback: CORRECT! Telecommunication companies are usually publicly listed entities, and
thus, their financial reporting is more stringent.
Score: 1 out of 1 Yes
Question 10
All of the following are examples of entities in the oil and gas industries’ upstream
sector, EXCEPT:
Response: Entities that transport (by pipeline, rail, barge, or truck) and store crude or refined
petroleum products.
Feedback: CORRECT! They are examples of entities operating in the oil and gas industries
midstream sector.

MANAGEMENT ADVISORY SERVICES

MANAGEMENT ACCOUNTING PART 1


Question 1
Jax Detergent is sold at a price of P100 per bottle with a variable cost of P20 per bottle. The
fixed expense of the business is P30,000 per year. Compute for the break-even point in units.
Response: 375
Feedback: CORRECT! The formula to compute the break-even point in units is fixed cost
divided by the contribution margin per unit. Substituting the computed values: P30,000/P80 =
375
Score: 1 out of 1 Yes
Question 2
Dantes Products applies fixed overhead at a rate of P3 per direct labor hour. Each unit
produced is expected to take two (2) direct labor hours. Dantes Products expected production
for the current year to be 10,000 units; however, only 9,000 units were actually produced.
Actual direct labor hours were 19,000 and actual fixed overhead costs were P62,000.
How much is the fixed overhead volume variance of Dantes Products?
Response: P6,000
Feedback:
CORRECT! This can be computed as follows:

Budget Overhead P60,000


Applied Fixed Overhead
54,000
(9,000 x 2 x P3)
Fixed Overhead Volume
P6,000
Variance - Unfavorable
Score: 1 out of 1 Yes
Question 3
The flower shop owner desires to achieve a profit of P50,000 by selling 1,000 bouquets of red
roses. The fixed cost involved in acquiring the red roses is P30,000, while the variable cost per
bouquet is P50. Determine the price per bouquet of red roses.
Response: P130
Feedback: CORRECT! The formula to compute for the price per bouquet is total number of
units produced and sold multiplied by price per unit = (variable cost per unit x total number of
units produced and sold) + fixed costs + desired profit. Substituting the given values: 1,000p =
(1,000) (P50) + P30,000 + P50,000; 1,000p=P130,000; p = P130.
Score: 1 out of 1 Yes
Question 4
Coffeebucks Company produces a single product. Last year, the company had 16,000 units in
its beginning inventory. During the year, the company’s variable production costs were P6 per
unit and its fixed manufacturing overhead costs were P4 per unit. The company’s net operating
income for the year was P24,000 higher under absorption costing than it was under variable
costing. Given these facts, what is the number of units in the ending inventory?
Response: 22,000 units
Feedback:
CORRECT! This can be computed as follows:

Beginning Inventory 16,000


Add: Change in the Inventory (P24,000/P4) 6,000
Ending Inventory in Units 22,000
Score: 1 out of 1 Yes
Question 5
The Hawaii Co. has made the following information available for its production facility for June
20X1. Fixed overhead was estimated at 19,000 machine hours for the production cycle. Actual
machine hours for the period were 18,900, which generated 3,900 units.
The following data are available:
 Material purchased (80,000 pieces), P314,000
 Material quantity variance, P6,400 – Unfavorable
 Machine hours used (18,900 hours)
 Variable overhead (VOH) spending variance, P50 – Unfavorable
 Actual fixed overhead, P60,000
 Actual labor cost, P40,120
 Actual labor hours, 5,900
Hawaii’s standard costs are as follows:
 Direct material, 20 pieces @ P4.00 per piece
 Direct labor, 1.5 hours @ P6.00 per hour
 Variable overhead (applied on a machine hour basis), P4.8 hours @ P2.50 per hour
 Fixed overhead (applied on a machine hour basis), 4.8 hours @ P3.00 per hour

How much is the conversion cost efficiency variance (CCEV)?


Response: P750 Unfavorable
Feedback:
CORRECT! This can be computed as follows:

Labor Efficiency Variance


P300 U
[5,900 – (3,900 x 1.5)] x P6
VOH Efficiency Variance
450 U
[18,900 – (3,900 x 4.8)] xP2.50
Conversion Cost Efficiency Variance P750 U

Score: 1 out of 1 Yes


Question 6
Which management function involves decisions relating to manufacturing methods and
marketing techniques?
Response: Strategic Management
Feedback: CORRECT! Strategic management also involves distribution channels, customer
profitability, and other long-term issues.
Score: 1 out of 1 Yes
Question 7
Viking Company produces a single product. During May, the company had net operating
income under absorption costing that was P3,500 lower than under variable costing. The
company sold 7,000 units in May, and its variable cost was P7 per unit, of which P3 was
variable selling expense. If fixed manufacturing overhead was P2 per unit under absorption
costing, then how many units did the company produce during May?
Response: 5,250 units
Feedback:
CORRECT! It can be computed as follows:

Number of units sold 7,000


Less: Change in Inventory (P3,500/2) 1,750
Units of Production 5,250

Score: 1 out of 1 Yes


Question 8
It pertains to a product, service, customer, etc. to which costs are accumulated for
management purposes.
Response: Cost object
Feedback: CORRECT! Cost objects can also be in the form of an activity or organizational unit.
Score: 1 out of 1 Yes
Question 9
XYZ Corp.’s actual data for 201A is given as follows:

Units produced 80,000


Units sold 76,000
Selling price per unit P600
Direct material costs 80
Direct labor 60
Variable manufacturing overhead 200
Variable selling costs 80
Fixed manufacturing costs P1,840,000
Fixed administrative costs 1,120,000
Compute for the net income under variable costing.
Response: P10,720,000
Feedback: CORRECT! The formula to compute for the net income under absorption costing is
contribution margin less fixed costs. Substituting the computed values: P13,680,000 -
P2,960,000 = P10,720,000
Score: 1 out of 1 Yes
Question 10
Santino is a well-known political analyst. He is the darling of the press.
RBS Publisher is negotiating to publish Santino’s Manifesto, a new book that has the potential
of being an instant bestseller. The fixed cost of producing and marketing the book will be
P800,000. The variable costs of producing and marketing will be P5.00 per copy sold. These
costs are before any payment to Santino.
Santino negotiates an upfront payment of P3.2 million, plus a 10% royalty rate on the net sales
price of each book. The net sales price is the listed book price of P35, less the margin to be
paid to in selling the book. The normal margin to be applied is 30% of the listed bookstore
price.
How many copies of Manifesto RBS Publisher must sell to earn a target operating income of
P3 million?
Response: 410,557
Feedback:

Selling Price 24.50


Less: Variable Cost 7.45
Contribution Margin/Unit 17.05

Fixed Cost 4,000,000.00


Add: Target Income 3,000,000.00
Total 7,000,000.00
Divide: Contribution Margin/Unit 17.05
Required Sales - Unit 410,557.18

Score: 1 out of 1

MANAGEMENT ACCOUNTING PART 2


Question 1
ABC Company manufactures one (1) product. Its sales price is expected to be P40 per unit.
Actual sales for November 201A are 1,550 units and 1,700 units for December 201A. ABC
budgets its sales for the next six (6) months for 201B: January – 675; February – 650; March –
688; April – 625.
All sales are on account. ABC collects its accounts as follows: 70% in the month of sale, 20%
in the month following the sale, and 10% in the second month following the sale. Uncollectible
accounts are negligible and can be disregarded. The beginning inventory on January 1, 201B
is 68 units. ABC desires an ending inventory of 10% of the next month’s budgeted
sales. Determine the sales budget for the first quarter of 201B.
Response: P80,520
Feedback: CORRECT! The formula to compute for the total budget is the sum of January to
March 201B sales multiplied by the selling price. Substituting the given values: (675x40) +
(650x40) + (688x40); 27,000 + 26,000 + 27,520 = P80,520
Score: 1 out of 1 Yes
Question 2
The management of Park John Corporation (PJC) has decided to implement a transfer pricing
system. PJC’s Management Information System (MIS) department is currently negotiating a
transfer price for its services with the four (4) producing divisions of the company and the
marketing department. Charges will be assessed based on the number of reports (assume that
all reports require the same amount of time and resources to produce).
The cost to operate the MIS department at its full capacity of 1,000 reports per year is
budgeted at P45,000. The user subunits expect to request 250 reports each this year. The cost
of temporary labor and additional facilities used to produce reports beyond capacity is
budgeted at P48.00 per report. PJC could purchase the same services from an external
Information Services firm for P70,000.
What amounts should be used as the ceiling and floor to determine the negotiated transfer
price?
Response: Floor: P45.60; Ceiling: P56.00
Feedback: CORRECT! Negotiated transfer prices should fall within a range limited by a ceiling
and a floor. The ceiling is the lowest market price that could be obtained from an external
supplier, and the floor equals the outlay costs plus the opportunity cost of the transferring
division. Since PJC’s MIS department does not have the option to sell services to external
customers, its opportunity cost is P0. Since the revenue-producing departments must cover all
costs of service departments, the MIS department’s outlay cost equals its total costs. The
department’s full capacity level is 1,000 reports per year. However, the user departments will
be requesting 1,250 reports (5 user subunits × 250 reports each). Thus, the MIS department
will incur costs of P12,000 [P48 × (1,250 – 1,000)] for the 250 reports above capacity, in
addition to the P45,000 budgeted costs for full capacity. The total cost of P57,000 (P45,000 +
P12,000) is used to calculate the floor. The ceiling is based on the P70,000 that would be
incurred to purchase MIS services externally. Since the MIS department will be producing
1,250 reports, the floor is P45.60 (P57,000 ÷ 1,250), and the ceiling is P56.00 (P70,000 ÷
1,250).
Score: 1 out of 1 Yes
Question 3
The balanced scorecard generally uses performance measures with four (4) different
perspectives. Which of the following performance measures would be part of those used for the
internal business processes perspective?
Response: Cycle time
Feedback: CORRECT! The cycle time is the time it takes to manufacture a product, and thus,
is an important part of the business processes perspective.
Score: 1 out of 1 Yes
Question 4
The following 20X1 information of Asahi Enterprise is available:
Pre-tax operating profit P60,000,000

Depreciation expense 15,000,000

Change in net working capital 10,000,000

Capital expenditures 12,000,000


Invested capital (Total assets – Current liabilities) 100,000,000
Current liabilities 20,000,000

Weighted – average cost of capital 10%

Income tax rate 40%

What is the amount of economic value added (EVA)?


Response: P26,000,000
Feedback:
CORRECT! This can be computed as follows:

After-tax operating profit (P60,000,000 x 60%) P36,000,000


Less: Cost of debt and capital (P100,000,000 x 10%) 10,000,000
Economic Value Added (EVA) P26,000,000
Score: 1 out of 1 Yes
Question 5
Truz Division of Teasen Corporation produces electric motors, 20% of which are sold to Teddy
Division of Teasen Corporation and the remainder to outside customers. Teasen Corporation
treats its divisions as profit centers and allows division managers to choose their sources of
sale and supply. Corporate policy requires all interdivisional sales and purchases to be
recorded at variable cost as a transfer price. Truz Division’s estimated sales and standard cost
data for the year ending December 31, 20X1, based on the full capacity of 100,000 units, are
as follows:

Teddy Outsiders
Sales P900,000 P8,000,000
Variable Costs (900,000) (3,600,000)
Fixed Costs (300,000) (1,200,000)
Gross Margin (P300,000) P3,200,000
Unit Sales 20,000 80,000

Truz has an opportunity to sell the above 20,000 units to an outside customer at P75 per unit
during 20X0 continuingly. Teddy can purchase its requirements from an outside supplier at P85
per unit.

Assuming that Truz desires to maximize its gross margin, should it take on the new customer
and drop its sales to Teddy for 20X1, and why?
Response: Yes, because Truz Division’s gross margin would increase by P600,000.
Feedback: CORRECT! If Truz sells to the new customer, its revenues will increase to
P1,500,000 (P75 × 20,000), but its costs will remain the same at P1,200,000 (P900,000 +
P300,000). This results in a positive gross margin of P300,000 (P1,500,000 – P1,200,000). The
new gross margin is P600,000 [P300,000 – (– P300,000)] greater than the original gross
margin.
Score: 1 out of 1 Yes
Question 6
The following 20X1 information of Asahi Enterprise is available:

Pre-tax operating profit P60,000,000


Depreciation expense 15,000,000
Change in net working capital 10,000,000
Capital expenditures 12,000,000
Invested capital (Total assets – Current liabilities) 100,000,000
Current liabilities 20,000,000
Weighted – average cost of capital 10%
Income tax rate 40%

What is the free cash flow (FCF) for 20X1?


Response: P29,000,000
Feedback:
CORRECT! This can be computed as follows:

After-tax operating profit (P60,000,000 x 60%) P36,000,000


Add: Depreciation 15,000,000
Less:
Change in net working capital 10,000,000
Capital expenditures 12,000,000
Free Cash Flow (FCF) P29,000,000
Score: 1 out of 1 Yes
Question 7
Which perspective of the balanced scorecard includes measures of profitability and market
value?
Response: Financial perspective
Feedback: CORRECT! Financial measures show the impact of the firm’s policies and
procedures on the firm’s current financial position and, therefore, its current return to the
shareholders.
Score: 1 out of 1 Yes
Question 8
A company has budgeted sales for 20X1:

1st Quarter 12,000


2nd Quarter 14,000
3rd Quarter 18,000
4th Quarter 16,000

The ending finished goods inventory for each month equals 25% of the next quarter’s budgeted
unit sales. Additionally, four (4) pounds of raw materials are required for each finished unit
produced. The ending raw materials inventory for each month equals 10% of the next month’s
production requirements. What is the budgeted or scheduled production for the 3rd Quarter?
Response: 17,500
Feedback:
CORRECT! This can be computed as follows:

Sales 18,000
Add: Ending Finished Goods Inventory 4,000
Less: Beginning Finished Goods Inventory (4,500)
Number of Units Produced 17,500
Score: 1 out of 1 Yes
Question 9
Clay Co. has considerable excess manufacturing capacity. A special job order’s cost sheet
includes the following applied manufacturing overhead costs:

Fixed costs P21,000


Variable costs 33,000

The fixed costs include a normal P3,700 allocation for in-house design costs, although no in-
house design will be done. Instead, the job will require the use of external designers costing
P7,750. What is the total amount to be included in the calculation to determine the minimum
acceptable price for the job?
Response: P40,750
Feedback: CORRECT! When determining a price for special order with idle capacity, only the
differential manufacturing costs are considered. The underlying assumption is that acceptance
of the order will not affect regular sales. In the short run, fixed costs are sunk costs and are
irrelevant. Since regular sales will not be affected by the special order, fixed and variable costs
incurred during normal operations are not considered. Clay Company should consider only the
variable costs associated with the order and the differential cost of using the external
designers. The costs to be considered total P40,750 (P33,000 + P7,750). The order is
accepted if revenue from the order exceeds the differential costs.
Score: 1 out of 1 Yes
Question 10
Lorreyn Company uses flexible budgeting for cost control. Lorreyn produced 10,800 units of
product during October, incurring indirect materials costs of P13,000. Its master budget for the
year reflected indirect materials costs of P180,000 at production volume of 144,000. What is
the indirect materials cost that shall be reflected in the October’s production flexible budget?
Response: P13,500
Feedback: CORRECT! The cost of indirect materials for 144,000 units was expected to be
P180,000. Consequently, the budgeted unit cost of indirect materials is P1.25
(P180,000/144,000). Multiplying the P1.25 to the 10,800 units is equal to P13,500 indirect
materials cost.
Score: 1 out of 1

FINANCIAL MANAGEMENT PART 1


Question 1
Harlo Company sells 20,000 radios evenly throughout the year. The cost of carrying one (1)
unit in inventory for one (1) year is P8, and the purchase order cost per order is P32. What is
the economic order quantity (EOQ)?
Response: 400
Feedback:
CORRECT! The EOQ formula is:

In the above equation, cost of placing one order, annual demand in units, and cost of carrying
one (1) unit in inventory for one (1) year. Substituting the given information, the equation
becomes

Score: 1 out of 1 Yes


Question 2
A firm has daily cash receipts of P100,000. A bank has offered to reduce the collection time of
the firm’s deposits by two (2) days for a monthly fee of P500. If money market rates are
expected to average 6% during the year, what is the net annual benefit (loss) from having the
bank’s service?
Response: P6,000
Feedback: CORRECT! The net benefit from the reduction of the cash receipts float is P12,000
[(P100,000 × 2) × 6%] minus the annual service fee, P6,000 (P500 × 12 months), which is
equal to P6,000.
Score: 1 out of 1 Yes
Question 3
Yomi Corporation has the following capital structure, which it considers to be optimal:

Debt 35%
Preferred Stock 5%
Common Equity 60%
100%

Yomi’s expected net income this year is P34,285.72, its established dividend payout ratio is
30%, its tax rate is 30%, and investors expect future earnings and dividends to grow at a
constant rate of 5%. Yomi paid a dividend of P2.40 per share last year, and its stock currently
sells for P48.00 per share. Yomi can obtain new capital in the following ways:

 New preferred stock with a dividend of P12.00 can be sold to the public at P80.00 per share.
 Debt can be sold at an interest rate of 10%.

What is the weighted average cost of capital (WACC) of Yomi Corporation?


Response: 10.51%
Feedback:
Correct! First, determine the cost of each capital component:

Cost of retained earnings:

Cost of preferred stocks:

Cost of debt:

Then, calculate the WACC:

Score: 1 out of 1 Yes


Question 4
Stock Y has the following historical returns for the past three (3) years:

Year Stock Y’s Returns


2x17 P36.81
2x18 P39.13
2x19 P28.01
What is Stock Y’s standard deviation on its historical returns?
Response: 5.87%
Feedback:
Correct! First, determine the expected return, which is the average return for the past three (3)
years.

Then, get the standard deviation.

Score: 1 out of 1 Yes


Question 5
Yesa Universal Music has P16 million in assets, and its tax rate is 40%. Its basic earning
power (BEP) ratio is 10%, and its return on assets (ROA) is 5%. What is its times-interest-
earned (TIE) ratio? Round off the final answer to whole number.
Response: 6.0
Feedback:
Correct! First, determine the Earnings Before Interest and Taxes (EBIT) using the formula of
BEP ratio.

Next, determine the Net Income using the Return on Assets (ROA) formula.
Then, get the Net Income Before Taxes (NIBT).

Then, get the difference of the EBIT and NIBT to determine the Interest Charges:

Lastly, get the Time-Interest Earned (TIE) Ratio:

Score: 1 out of 1 Yes


Question 6
The real risk-free interest rate is 2.5%, and it is expected to remain constant over time. Inflation
is expected to be 2% per year for the next four (4) years and 3% per year for the next five (5)
years. The maturity risk premium is equal to 0.2 x (t - 1)%, where t equals the bond’s maturity.
The default risk premium for a Baa-rated bond is 1.1%.
What is the quoted interest rate on a five-year Baa-rated corporate bond with a liquidity
premium of 0.4%?
Response: 7%
Feedback:
Correct! First, determine the applicable inflation rate.
Then, solve for the quoted interest rate:

Score: 1 out of 1 Yes


Question 7
In The Philippine Stock Exchange (PSE) Journal, you read that 30-day T-bills are currently
yielding 5.8%. Your sister, a broker at S&P Global, has given you the following estimates of
current interest rate premiums:
 25% Inflation premium (IP)
 6% Liquidity premium (LP)
 85% Maturity risk premium (MRP)
 15% Default risk premium (DRP)
Based on these data, what is the real risk-free rate (r*) of return?
Response: 1.7%
Feedback:
Correct! In a risk-free bond such as Treasury Bills, there are only three (3) factors to consider:
risk-free rate, inflation premium, and maturity risk premium.
Thus,

Then, to derive the real-risk free rate

Score: 1 out of 1 Yes


Question 8
Teasen Company has P2,392,500 in current assets and P1,076,625 in current liabilities. Its
initial inventory level is P526,350, and it will raise funds as additional short-term notes payable
and use them to increase inventory. How much can its short-term debt (notes payable)
increase without pushing its current ratio below 2.0?
In computing for the percentage, do not round off. Round off the final answer to whole number.
Response: P239,250
Feedback:
Correct! First, determine the current ratio before the increase:

Then compute for the required increase.

Score: 1 out of 1 Yes


Question 9
Teasen Merchandise has P2.5 million in inventory and P2 million in accounts receivable. Its
average daily sales are P100,000. The firm’s payables deferral period is 30 days, and the
average daily cost of sales is P50,000. What is the length of the firm’s cash conversion period?
Response: 40 days
Feedback: CORRECT! The cash conversion period is calculated as the inventory conversion
period + receivables collection period – payables deferral period. Applying the aforementioned
formula, the inventory conversion period is P2,500,000/P50,000 = 50 days, and the receivable
conversion period is P2,000,000/P100,000 = 20 days. Therefore, the cash conversion cycle
equals 50 days + 20 days – 30 days = 40 days.
Score: 1 out of 1 Yes
Question 10
Stay Corporation has earnings before interest and taxes (EBIT) of P20,000,000, depreciation
of P4,000,000, and a 30% tax rate. It needs to spend P12,000,000 on new fixed assets and
another P12,000,000 to increase its current assets. It expects its accounts payable to increase
by P3,000,000, accruals to increase by P3,500,000, and notes payable to increase by
P5,000,000. The firm’s current liabilities consist of accounts payable, accruals, and notes
payable. What is the company’s free cash flow (FCF)?
Response: P500,000
Feedback:
Correct! Determine the change in the net operating working capital (NOWC):

Solve for the free cash flow (FCF).

FINANCIAL MANAGEMENT PART 2


Question 1
Young Corp. desires to purchase equipment for its production process. The equipment costs
P300,000 and would result in the following cash inflow: Year 1 – P90,000; Year 2 – P75,000;
Year 3 – P55,000; Year 4 – P50,000. Decide whether Young Corp. should purchase the
equipment using the concept of net present value (NPV) under a discount rate of 12%.
Response: Reject
Feedback: CORRECT! After computing the NPV, the answer would be -P88,929.51, less than
zero. Based on the criterion of NPV, the decision would be to reject the purchase of the
equipment.
Score: 1 out of 1 Yes
Question 2
After several years of producing and selling at 100,000 units, GRG Company faced a year with
projected sales and production of 76,000 units. A potential customer offered to purchase
14,000 units at a price of P36 each. The normal sales price is P60 each.
The following data are available: Direct material – P18.00; Direct labor – P13.00; Variable
manufacturing overhead – P4.00; Fixed manufacturing overhead – P7.50.
Should GRG accept or reject the offer?
Response: Given the excess capacity, GRG Company should accept the offer because it
increases the income by P14,000.
Feedback: CORRECT! The formula for the incremental income is contribution margin (selling
price – relevant costs) x units sold. Substituting the given values: (P36- P35) x 14,000 =
P14,000
Score: 1 out of 1 Yes
Question 3
Line Corporation (LC) currently manufactures part B137, producing 80,000 units annually. The
part is used to produce several products made by the company. The cost per unit for B137 is
as follows: Direct material – P18.00; Direct labor – P6.00; Variable manufacturing overhead –
P5.00; Fixed manufacturing overhead – P8.00.
Of the total fixed overhead assigned to B137, P176,000 is avoidable (the lease of production
machinery and salary of a production line supervisor–neither of which will be needed if the line
is dropped). The remaining fixed overhead is a common fixed overhead. An outside supplier
has offered to sell the part to LC for P32. There is no alternative use for the facilities currently
used to produce the part.
Should LC make or buy part B137?
Response: It is better to make the parts because the cost to produce is lower than buying the
parts from a supplier. It results in a P64,000 net advantage to make.
Feedback: CORRECT! Adding the relevant costs in the “make” decision gives cost savings for
the firm.
Score: 1 out of 1 Yes
Question 4
Jun Company wants to invest in a machine costing P80,000 with a useful life of six (6) years
and no salvage value. The machine will be depreciated using the straight-line method and is
expected to produce an annual cash inflow from operations of P22,000, net of income taxes.
The present value of an ordinary annuity of P1 for six (6) years at 10% is 4.355. The present
value of P1 for six (6) periods at 10% is 0.564. What is the net present value (NPV) of the
investment?
Response: P15,810
Feedback:
CORRECT! This can be computed as follows:

Present value of annual cash inflows for six


(6) periods at 10% (P22,000 x 4.355) P95,810
Less: Present value of the net investment 80,000
Net present value P15,810
Score: 1 out of 1 Yes
Question 5
Which theory is useful in explaining the general movement of interest rates for a particular
country?
Response: Loanable Funds
Feedback: CORRECT! This theory assumes that the higher the interest rates, sectors in the
market will be more willing to supply loanable funds; the lower the level of the interest, the less
they are willing to provide.
Score: 1 out of 1 Yes
Question 6
ABC Corp. plans to borrow P1,000,000 funds from XYZ Inc. The risk-free rate imposed on the
loan is 5%. XYZ’s debt margin is 3%. How much interest rate should XYZ Inc. impose on ABC
Corp.?
Response: 8%
Feedback: CORRECT! The formula to compute for the interest rate is i=Rf+ Dm; Substituting the
given values: I = 5%+ 3% = 8%
Score: 1 out of 1 Yes
Question 7
Which yield curve can be observed when bond investors expect the economy to grow at a
normal pace, without significant changes in the rate of inflation or major interruptions in
available credit?
Response: Upward
Feedback: CORRECT! This is the usual flow of the slope, which means that the long-term
interest rates are above the short-term interest rates.
Score: 1 out of 1 Yes
Question 8
A 1-year treasury bill has a coupon rate of 2%, and a 2-year treasury note has a coupon rate
of 3%. Based on the pure expectations theory, what is the 1-year interest rate anticipated
at t +1 (2nd year)?
Response: 4.00980%
Feedback: CORRECT! The formula to compute for the interest rate is (1+ ti2 )2 = (1+tii) (1+ t+1r1);
Substituting the given values: (1.03)2= (1.02) (1+x). Then, finding the value of x: 1.0609 = 1.02
+ 1.02x = 4.00980%
Score: 1 out of 1 Yes
Question 9
Jin Company (JC) plans to spend P60,000 on a machine that will be depreciated on a straight-
line basis over 10 years. The machine will generate additional cash revenues of P12,000 a
year. JC will not incur additional costs except for depreciation. The income tax rate is 35%.
What is the payback period of the said investment?
Response: 6.06 years
Feedback:
CORRECT! This can be computed as follows:

Score: 1 out of 1 Yes


Question 10
Yessa Corporation is considering replacing its old machine with a book value of P150,000 and
a remaining useful life of three (3) years. The old machine will be replaced with a new one that
will cost P375,000, with a three-year useful life and no salvage value.
The annual operating costs of the old machine amount to P180,000, which can be reduced by
55% if a new machine is acquired. The old machine will have no disposal value after three (3)
years but can be disposed of now at P60,000. How much is the differential cost between the
alternatives?
Response: P18,000
Feedback:
CORRECT! This can be computed as follows:

Cost of the new machine P375,000


Add: Operating costs (P180,000 x 45% x3) 243,000
Less: Disposal value of old machine (60,000)
Total relevant cost to replace P558,000
Total relevant cost to retain (P180,000 x 3) 540,000
Differential cost P18,000
Score:

MANAGEMENT CONSULTANCY AND REPORTING


Question 1
Which type of pricing objective ignores market share and focuses on the price where profit is
enlarged?
Response: Profit maximization
Feedback: CORRECT! Profit maximization is a business practice wherein firms determine the
output price levels that lead to the highest profit, regardless of customers’ price preference.
Score: 1 out of 1 Yes
Question 2
Risa Bakery is known for its tasty pastries. During the first month of production, it sold 750
cinnamons at the price of P60. Increasing demand encouraged them to supply 1,500 in the
following month at the price of P80. Calculate the price elasticity of supply.
Response: 3.03
Feedback:
CORRECT! The formula to compute for the given problem is
Score: 1 out of 1 Yes
Question 3
Which of these is NOT included in disequilibrium profit theories?
Response: Oligopoly profit
Feedback: CORRECT! Oligopoly is a market structure characterized by a small number of
large firms. However, there is no such theory about oligopoly profit.
Score: 1 out of 1 Yes
Question 4
Kim Sy Merchandising is the major automotive parts supplier in a local town. As the company
became well-known, it started increasing the price of its best-selling automotive accessory from
P2,000 to P2,500. As a result, the quantity of their product sold fell from 700 to 500. Using the
concept of price elasticity, compute for the percent (%) change in quantity.
Response: 33.33%
Feedback:
CORRECT! The formula to compute for the given problem is

Where:
= Current quantity
= Previous quantity

This means that the percent change in the quantity of Kim Junkyu Merchandising’s best-selling
automotive accessory fell to 33.33%.
Score: 1 out of 1 Yes
Question 5
Jung Merchandising is the major supplier of automotive parts in a local town. As the company
became well-known, it started increasing the price of its best-selling automotive accessory from
P2,000 to P2,500. As a result, the quantity of their product sold fell from 700 to 500. Determine
the demand elasticity of the given product.
Response: Elastic
Feedback: CORRECT! The given product is considered elastic because the quotient, derived
from the price elasticity, is 1.5 and is, therefore, greater than one (1).
Score: 1 out of 1 Yes
Question 6
This happens when a firm produces the maximum output possible for a given combination of
inputs and existing technology.
Response: Technical efficiency
Feedback: CORRECT! When a firm is technically efficient, every input is being utilized to the
fullest extent possible. There is no other way to get more output without using more of at least
one (1) input.
Score: 1 out of 1 Yes
Question 7
Which market structure is characterized by a large number of firms producing identical
products?
Response: Perfect competition
Feedback: CORRECT! It has the highest degree of competition due to the large number of
firms producing identical products in perfect competition.
Score: 1 out of 1 Yes
Question 8
Jeff Batambakal receives an income of P3,000 per month and spends P2,500. Jeff’s income
increased by P500 per month, and he now spends P2,800 per month. What is Jeff’s marginal
propensity to save?
Response: 0.40
Feedback:
CORRECT! To solve for the marginal propensity to save, compute first for the change in
savings which is P200 [(P3,500 – P2,800) – (P3,000 – P2,500)]. Afterward, divide the change
in savings (P200) with the change in income of P500.

Score: 1 out of 1 Yes


Question 9
Which type of pricing objective aims to maximize sales while making a normal profit or break-
even profit?
Response: Sales maximization
Feedback: CORRECT! Sales maximization involves selling at a price equal to the average
cost.
Score: 1 out of 1 Yes
Question 10
Kaye Jose has the following data:

Level of
Disposable
Consumption
Income
P42,000 P36,000
P50,000 P42,000

What is Kaye Jose’s marginal propensity to consume?


Response: 0.75
Feedback: CORRECT! To solve the marginal propensity to consume, compute for the change
in consumption, P6,000 (P42,000 – P36,000). Afterward, divide it with the change in the
disposable income of P8,000 (P50,000 – P42,000).

ADVANCED FINANCIAL ACCOUNTING AND REPORTING


ACCOUNTING FOR SPECIAL TRANSACTIONS 1
Question 1
Using the information in CASE NO. 1 (b), what amount of these earnings should be credited to
each partner’s capital account?
Response: Red – P43,000; White – P37,000
Feedback:
CORRECT! Credits to partners’ capital accounts are based on earnings after allowance for
interest, salary, and bonus. The earnings before any allowance of P80,000 are reduced by the
salary allowances of P100,000 and result in a loss of P20,000.

The P20,000 loss is then distributed to the partners in relation to their profit and loss ratios as
follows:

Red: P55,000 – (P20,000 x 60%) = P43,000


White: P45,000 – (P20,000 x 40%) = P37,000
Score: 1 out of 1 Yes
Question 2
Using the information in CASE NO. 2, how much is the estimated deficiency to creditors?
Response: P36, 000
Feedback:
CORRECT! The estimated deficiency is computed by getting the difference between
the net free assets (not the total) and the total unsecured liabilities without priority.
Net free assets = P160,000 – P14,000 = P144,000
Unsecured liabilities without priority = P25,000 + P155,000 = P180,000
Estimated deficiency = P144,000 – P180,000 = (P36,000)
Score: 1 out of 1 Yes
Question 3
Using the information in CASE NO. 2, what amount is paid to unsecured creditors without
priority?
Response: P124,000
Feedback:
CORRECT! The estimated recovery percentage is computed by dividing the net free assets by
the total unsecured liabilities.

P144,000/P180,000 = 80%

P155,000 x 80% = P124,000


Score: 1 out of 1 Yes
Question 4
Using the information in CASE NO. 3 (b), what is the amount of sales revenue that Patrick
Company must report on its interest in Loresco Company?
Response: P1,440,000
Feedback:
CORRECT! The amount can be computed as follows:

Score: 1 out of 1 Yes


Question 5
Using the information in CASE NO. 1 (c), how much is the share of A in the P28,000 cash for
distribution, assuming that any partner’s capital debit balance is uncollectible?
Response: P17,800
Feedback:
CORRECT! This is the cash payment to Partner A.
A (3/6) B (2/6) C (1/6)
Bal before liquidation P40,000 P25,000 P5,000
Loss on realization (P42,000) 21,000 (14,000) (7,000)
Balance P19,000 P11,000 (2,000)
Absorption of C (3:2) (1,200) (800) 2,000
Cash payment P17,800 P10,200 P0

Score: 1 out of 1 Yes


Question 6
Using the information in CASE NO. 3 (a), what is the investment income that Entity M must
report for the year ended December 31, 20X1?
Response: P1,182,000
Feedback:
CORRECT! The amount can be computed as follows:

Score: 1 out of 1 Yes


Question 7
Using the information in CASE NO. 1 (a), what amount should Junkyu’s capital account
change?
Response: P7,000 increase
Feedback:
CORRECT! Before distributing the net loss, the interest and salaries should be allocated first
because this will also increase the amount of loss.

This increased loss would then be allocated to the partners, decreasing their capital accounts.
The computations are shown below.

Junkyu Yedam Jeongwoo

10% interest on average


P12,000 P6,000 P4,000
capital

Salaries 30,000 20,000

Balance (equally) (35,000) (35,000) (35,000)


Total increase (decrease) P7,000 (P29,000) (P11,000)

Score: 1 out of 1 Yes


Question 8
Using the information in CASE NO. 3 (a), what is the balance of Investment in Entity O that
Entity N must report on December 31, 20X1?
Response: P2,672,000
Feedback:
CORRECT! The amount can be computed as follows:

Score: 1 out of 1 Yes


Question 9
Using the information in CASE NO. 1 (d), what is the cash available for distribution to M after
payment of liabilities?
Response: P24,000
Feedback:
CORRECT! This amount can be computed using the safe payment schedule. The safe
payment schedule should start with:
 Computation of the total interest of the partners at the beginning of an installment period.
 The gain or loss on realization of non-cash assets should then be allocated to each of the
partners’ total interest using the P/L ratio.
 The maximum possible loss will then be deducted from the balance after allocating gain or loss
on realization.
 After allocating maximum possible loss, the first installment payments to the partners are
determined.

M (40%) N (30%) O (30%)

Capital balances P60,000 P40,000 P40,000


Loss on realization
(16,000) (12,000) (12,000)
(P40,000)
Maximum possible loss
(20,000) (20,000) (15,000)
(P130,000 – P80,000)
Cash distribution P24,000 P13,000 P13,000
Score: 1 out of 1 Yes
Question 10
Using the information in CASE NO. 3 (b), what is the value of the total liabilities that Rachelle
Company must report on its interest in Loresco Company?
Response: P3,800,000
Feedback:
CORRECT! The amount can be computed as follows:

Score: 1 out of 1
ACCOUNTING FOR SPECIAL TRANSACTIONS II
Question 1
Using the information in CASE NO. 5 (a), what is the net income?
Response: P23,000
Feedback:
CORRECT! The freight costs to be included as the cost of chillers sold shall pertain only to the
sold units. Therefore, only P2,000 shall be included.

Sales, P100,000 – Cost of chillers sold, P60,000 + P2,000 freight cost = Gross profit, P38,000
– Commission expense, P15,000 = P23,000 net income
Score: 1 out of 1 Yes
Question 2
Using the information in CASE NO. 6, how much loss should Kaiser recognize in 20x1?
Response: P50,000
Feedback: CORRECT! When it is probable that the contract costs will exceed total contract
revenue, the expected loss should be recognized as an expense immediately. Instead of
recognizing P15,000 only, the anticipated loss shall be fully recognized. (P15,000/30%
=P50,000)
Score: 1 out of 1 Yes
Question 3
Using the information in CASE NO. 4 (a), what is the unearned franchise revenue for the year
ended December 31, 20x1?
Response: P119,700
Feedback:
CORRECT! This is the unearned revenue from the license. Only P13,300 is considered earned
during 20x1 since the amount allocated to the license of P133,000 will be amortized over time
for 10 years.

Therefore, P119,700 is still unearned for the year ended December 31, 20x1.
Score: 1 out of 1 Yes
Question 4
Using the information in CASE NO. 5 (b), what are the total collections during 20x2 (including
collections for 20x1)?
Response: P300,000
Feedback:
CORRECT! The total collections for 20x2 includes P150,000 each for 20x1 and 20x2
installment sales.

T-accounts:

Inst. Receivable, 20x1


IS 300,000 90,000 Coll. Yr. 1
End 210,000
150,000 Coll. Yr.
2
End, 20x2 60,000

DGP, end of 15,000/25% = P60,000 IR, end.

Inst. Receivable, 20x2


IS 375,000 150,000 Coll. Yr. 2
End 225,000

DGP, end of 54,000/24% = P225,000 IR, end.

Total collections = P150,000 + P150,000 = P300,000


Score: 1 out of 1 Yes
Question 5
Using the information in CASE NO. 5 (a), what amount should Elyssa Inc. recognize as
consignment sales revenue for 20x1?
Response: P100,000
Feedback:
CORRECT! The sales revenue is computed by multiplying the number of units sold by the
selling price per unit.

10 units x P10,000 = P100,000


Score: 1 out of 1 Yes
Question 6
Using the information in CASE NO. 4 (b), how much is the franchise revenue on November 1,
20x1, assuming that the franchisor requires no future services once the franchisee begins
operations?
Response: P83,110
Feedback:
CORRECT! This is the franchise revenue recognized on November 1, 20x1.

Cash down payment P80,000

PV of note (P60,000 x 1.7833) P106,998

Total transaction price P186,998

Ratio Revenue
Rights to trade
80/180 P83,110
name
Training services 23/180 -
Equipment 77/180 -

Training services and equipment are separately classified and not as an unearned franchise
revenue.
Score: 1 out of 1 Yes
Question 7
Using the information in CASE NO. 7 (b), what is the amount of inventory of the branch on
January 1, 20x2 insofar as the home office is concerned?
Response: P12,500
Feedback: CORRECT! Insofar as the home office is concerned, the amount to be reported
should be restated to its original costs. The amount of inventory on January 1, 20x2, at cost is
P12,500 computed as P15,625/1.25 = P12,500.
Score: 1 out of 1 Yes
Question 8
Using the information in CASE NO. 7 (a), how much is the branch's net income (loss) insofar
as the home office is concerned?
Response: P16,500
Feedback:
CORRECT! Note that the true profit is computed by simply restating the merchandise received
from the home office to their original costs. Alternatively, the true profit of the branch can be
calculated by adding the realized markup to the individual profit. The realized markup is the
markup on the shipments sold to external parties.

P303,050 – P61,050 = P242,000 x 10/110 = Realized markup, P22,000 – Individual net loss,
P5,500 = P16,500 True net income
Score: 1 out of 1 Yes
Question 9
Using the information in CASE NO. 5 (c), what is the gain or loss on repossession assuming
the repossessed merchandise is to be recorded at fair value?
Response: P1,100 loss
Feedback:
CORRECT! Since the unrecovered cost amounting to P5,600 (P8,000 x 70%) is greater than
the fair value of P4,500, there is a loss on repossession of P1,100.

Journal entry:
Inventory 4,500
DGP 2,400
Loss on Repossession 1,100
Installment Receivable 8,000
Score: 1 out of 1 Yes
Question 10
Anna Company (AC) provides furniture to Veronica Company (VC) on a consignment basis.
The products are immediately proposed for sale in the garden center of VC. AC retains title to
the products until they are sold to the end customer. VC does not have an obligation to pay AC
until a sale occurs, and any unsold products can be returned to AC. AC also retains the right to
take back any unsold products or transfer unsold products to another retailer. Once VC sells
the products to the end customer, AC has no further obligations, and the retailer has no further
return rights. Under Philippine Financial Reporting Standard (IFRS) 15, when shall AC
recognize revenue for the consignment arrangement?
Response: When VC sells the products to the end customer.
Feedback: CORRECT! IFRS 15 provides that revenue shall be recognized only when control is
passed. In the given case, VC only assigns AC the right to sell the products. Revenue will only
be recognized when VC sells the product to the customers.
ACCOUNTING FOR BUSINESS COMBINATIONS, CONSOLIDATIONS, AND FOREIGN
CURRENCY TRANSACTIONS
Question 1
Using the information in CASE NO. 10 (b), how much is the translated comprehensive income
of Samuel Company?
Response: P40,950
Feedback: CORRECT! The rate to be used in translating revenues and expenses is the
weighted average rate.
Score: 1 out of 1 Yes
Question 2
Using the information in CASE NO. 8, what is the value of the total assets that will appear in
the consolidated statement of financial position at the date of acquisition?
Response: P7,545,000
Feedback:
CORRECT! To solve for the total assets that will appear in the consolidated statement of
financial position at the date of acquisition, consolidate the book values of the parent’s assets
and the fair values of the subsidiary’s assets.

Thus, in the given case, the total consolidated assets as of the acquisition date is P7,545,000,
computed as follows:

Score: 1 out of 1 Yes


Question 3
Using the information in CASE NO. 10 (a), how much is the net forex gain or loss from the
importation?
Response: P30,000 loss
Feedback: CORRECT! To get the total net forex gain or loss in importing goods, get the
difference between the foreign currency's initial and settlement values.
Score: 1 out of 1 Yes
Question 4
Using the information in CASE NO. 9 (b), what is the consolidated comprehensive income
attributable to Park Corporation for the year 20X2?
Response: P89,400
Feedback: INCORRECT! The parent company's share from the subsidiary’s net income must
be added.
Score: 0 out of 1 No
Question 5
Using the information in CASE NO. 10 (b), how much is the translation adjustment for the
Statement of Financial Position of Samuel Company?
Response: P4,050
Feedback: CORRECT! The translation adjustment equals the difference of the total assets and
total liability and equity after translation.
Score: 1 out of 1 Yes
Question 6
Using the information in CASE NO. 9 (c), what is the value of the non-controlling interest on
December 31, 20X1?
Response: P393,000
Feedback:
CORRECT! The amount can be computed as follows:

Score: 1 out of 1 Yes


Question 7
Using the information in CASE NO. 8, what is the goodwill or gain from bargain purchase from
the business combination?
Response: P221,000 gain on bargain purchase
Feedback:
CORRECT! To solve for the goodwill or gain on bargain purchase, one shall determine the
difference between the aggregate amount of the consideration transferred, non-controlling
interest (NCI), and previously held equity over the fair value of the acquiree’s identifiable net
assets at the date of acquisition. If the aggregate amount is greater than the identifiable assets,
it is goodwill; otherwise, it is a “gain on bargain purchase.”

Applying the above concept, the business combination resulted in a P221,000 gain on bargain
purchase, computed as follows:

Score: 1 out of 1 Yes


Question 8
Using the information in CASE NO. 9 (a), how much is the additional paid-in capital that will
appear in the consolidated financial statement as of the acquisition date?
Response: P1,100,000
Feedback:
CORRECT! Upon consolidation, the equity accounts of the subsidiary are eliminated against
the investment in the subsidiary account. Thus, the equity in the consolidated statement of
financial position reflects those from the parents only.
In the given case, the P1,100,000 can be computed by getting the sum of the P500,000
original additional paid-in capital of the parents plus the additional P600,000 [30,000 shares x
(P30-P10)] paid-in capital upon issuance of new shares as consideration.
Score: 1 out of 1 Yes
Question 9
Using the information in CASE NO. 9 (a), how much is the total assets and liabilities that will
appear in the consolidated financial statement as of the acquisition date?
Response: P3,815,000
Feedback: CORRECT! The value of the total consolidated total assets is equal to the book
value of the parent’s total assets plus the acquisition-date fair values of the subsidiary.

Score: 1 out of 1 Yes


Question 10
Using the information in CASE NO. 9 (c), what is the consolidated net income attributable to
Pepito Company?
Response: P616,000
Feedback:
CORRECT! The amount can be computed as follows:

ACCOUNTING FOR GOV’T AND NOT FOR PROFIT ORG


Question 1
Using the information in CASE NO. 12 (e), what amount should the Gawad Kalinga report as
pledges receivable, net of any required allowance account?
Response: P360,000
Feedback:
CORRECT! The amount can be computed as follows:

Score: 1 out of 1 Yes


Question 2
Using the information in CASE NO. 12 (b), what amount should TU include in its unrestricted
funds as revenues from tuition fees?
Response: P350,000
Feedback: INCORRECT! The student grant is just an appropriation of the revenue from the
unrestricted fund.
Score: 0 out of 1 No
Question 3
Using the information in CASE NO. 11 (b), what is the journal entry to record the payment of
the materials and labor to construct the bridges?
Response: Debit to Construction in Progress – Infrastructure Assets, P100,000,000
Feedback: CORRECT! The Cash–Modified Disbursement System, Regular account shall also
be credited to record the payment of the materials and labor for the construction.
Score: 1 out of 1 Yes
Question 4
Using the information in CASE NO. 12 (a), what is the journal entry to record the purchase of
the laboratory equipment?
Response: Credit to Cash – Modified Disbursement System, Regular, P2,060,000
Feedback: CORRECT! Only the purchase price and delivery cost shall be directly credited to
the Cash – Modified Disbursement System, Regular account. The present value of the
dismantling cost shall be credited to the Other Provisions account.
Score: 1 out of 1 Yes
Question 5
Using the information in CASE NO. 12 (c), what amount of the receipts would be included in
the net cash provided (used) by operating activities on December 31,20X1?
Response: P1,500,000
Feedback:
CORRECT! The amount can be computed as follows:

Score: 1 out of 1 Yes


Question 6
Using the information in CASE NO. 11 (a), what is the journal entry to record the receipt of the
CDC?
Response: Credit to Subsidy from National Government, P15,000,000
Feedback: CORRECT! The Cash – Constructive Income Remittance account shall also be
debited to record the receipt of the CDC.
Score: 1 out of 1 Yes
Question 7
Using the information in CASE NO. 11 (e), what is the entry to be made by the municipality
upon the receipt of its share?
Response: Debit: Cash Local Treasury, P48,000; Credit: Real Property Tax - Basic, P48,000
Feedback: CORRECT! Since Mr. Reyes did not pay within the 10-day discount period, no
discount shall be deducted from the payment of RPT. Thus, the municipality will receive 40% of
the P120,000.
Score: 1 out of 1 Yes
Question 8
Using the information in CASE NO. 12 (d), which of the following entries will be reflected in the
plant fund ledger account?
Response: A credit to Mortgage Notes Payable, P600,000
Feedback: CORRECT! The Building account shall be debited for P1,500,000.
Score: 1 out of 1 Yes
Question 9
Using the information in CASE NO. 11 (d), how much is the balance of the deferred service
revenue at the end of 20X1?
Response: P792,000
Feedback: CORRECT! The deferred service revenue at the end of 20X1 equals the total
amount of construction cost, less the amount recognized for the year.
Score: 1 out of 1 Yes
Question 10
Using the information in CASE NO. 11 (c), what is the entry that shall be made by the
Department of Education on July 10, 20X1?
Response: A credit to Cash – Modified Disbursement System, Regular, P282,200
Feedback:
CORRECT! It is equal to the amount of the net payment. The net payment can be computed as
follows:

Accounts Payable P425,000


Less: Retention Fee 50,000
Withholding Tax 29,750
Net Amount 345,250
COST ACCOUNTING AND OTHER SPECIAL TOPICS
Question 1
Using the information in CASE NO. 13 (d), what is the predetermined rate based on direct
labor hours?
Response: P4.05
Feedback: Correct! Estimated MOH / DL Hours; P405,000.00 / 100,000 DLH = P4.05 per DLH
Score: 1 out of 1 Yes
Question 2
Using the information in CASE NO. 13 (b), which of the following is the cost of goods
manufactured?
Response: P646,500
Feedback: Correct! Direct Materials + Direct Labor + Overhead = Total Manufacturing Costs +
Work in Process, January 01 = Total Costs Put into Process - Work in Process, January 31 =
Cost of Goods Manufactured; P243,000 + P180,000 +P111,000= P534,000 + P225,000 =
P759,000 - P112,500 = P646,500
Score: 1 out of 1 Yes
Question 3
Using the information in CASE NO. 13 (d), what is the predetermined rate based on the units
of production?
Response: P4.05
Feedback: Incorrect! Predetermined rate based on direct labor hours
Score: 0 out of 1 No
Question 4
Using the information in CASE NO. 13 (c), which of the following is the total variable cost of
the highest level of activity?
Response: P2,933.14
Feedback: Incorrect! The total variable costs of the lowest level of activity
Score: 0 out of 1 No
Question 5
Using the information in CASE NO. 13 (a), which of the following is the cost of indirect
materials?
Response: P20,700
Feedback: Correct! 10% of the issued raw materials; P207,000x 10% = P20,700
Score: 1 out of 1 Yes
Question 6
Using the information in CASE NO. 13 (b), which of the following is the cost of goods sold?
Response: P691,500
Feedback:
Correct! Finished Goods, January 01 + Cost of Goods Manufactured = Total Goods Available
for Sale - Finished Goods, January 31 =
Cost of Goods Sold; P180,000 + P646,500 = P826,500 - P135,000 = P691,500
Score: 1 out of 1 Yes
Question 7
Using the information in CASE NO. 13 (e), what is the amount to be reported as the physical
units to be accounted for?
Response: 617,100
Feedback: Correct! Units completed and transferred to FG Inventory + Units in ending WIP
Inventory = Units accounted for; 609,000 + 8,100 = 617,100
Score: 1 out of 1 Yes
Question 8
Using the information in CASE NO. 13 (a), Which of the following is the raw materials
inventory on January 31?
Response: P172,500
Feedback: Correct! Raw Materials Inventory, January 01 + Purchases + Raw Materials Used =
P57,500 + P230,000 + P92,000 - P207,000 = P172,500
Score: 1 out of 1 Yes
Question 9
Using the information in CASE NO. 13 (c), which of the following is the total variable cost of
the lowest level of activity?
Response: P2,933.14
Feedback: Correct! 287 machine hours x PHP 10.22 = PHP 2,933.14
Score: 1 out of 1 Yes
Question 10
Using the information in CASE NO. 13 (e), what is the amount to be reported as the total
physical units to be accounted for?
Response: 617,100
Feedback: Correct! Units in beginning WIP Inventory + Units started during current period =
Units to account for; 15,000 + 602,100 = 617,100
Score: 1 out of 1
RFBT
LAW ON BUSINESS TRANSACTIONS
Question 1
Solidary debtors A, B, C, and D are obliged to give solidary creditors V, W, X, Y, and Z,
P20,000. Which is TRUE?
Response: V may collect P20,000 from A.
Feedback: CORRECT! This is true because there is mixed solidarity. In a solidary obligation,
each debtor is liable for the whole obligation, and each creditor is entitled to demand payment
of the whole obligation. Therefore, V may go after A and demand the payment of P20,000.
Score: 1 out of 1 Yes
Question 2
M, an insane person, orally sold shabu at a price of P1,000.00, although its book value is
P1,500.00, to I, an insane person. What is the status of the contract?
Response: Void
Feedback: CORRECT! It is void because the object of the contract is illegal.
Score: 1 out of 1 Yes
Question 3
What is the status of the sale of a specific house at a price of P1 million payable in 10 annual
installments when the delivery of the land will happen after three (3) years, but the buyer had
already paid P100,000 earnest money or down payment at the time of perfection?
Response: Perfectly valid
Feedback: CORRECT! The contract is already partially executed because earnest money was
paid at the time of perfection.
Score: 1 out of 1 Yes
Question 4
The following are examples of a quasi-contract, EXCEPT:
Response: A person takes charge of the agency or management of business or property of
another with the authority and consent of the latter.
Feedback: CORRECT! For a quasi-contract to exist, in this case, negotiorum gestio, a person
must voluntarily take charge of the agency or management of the business or property of
another without any power or authority from the latter.
Score: 1 out of 1 Yes
Question 5
A, a minor, sold in writing his ball pen for P1,000 to B, a capacitated person. The book value of
the ball pen is P1,500, so A suffered lesion by more than ¼ of the value of the thing, which is
the object thereof. What is the status of the contract?
Response: Voidable
Feedback: CORRECT! The contract is voidable because a minor is an incapacitated person.
This contract is more defective than rescissible contract. Defective contracts are arranged,
presented, and regulated in ascending order of defectiveness: (1) Rescissible (Least
defective); (2) Voidable; (3) Unenforceable; (4) Void (Most defective).
Score: 1 out of 1 Yes
Question 6
Jacob pledged his ring to secure his debt to Esau, amounting to P20,000, payable after 30
days. On the due date, Jacob defaulted. At a public auction, the ring was sold only for P18,000.
Which of these is TRUE?
Response: The pledge is extinguished; the debt will be extinguished when Esau has recovered
the deficit of P2,000 from Jacob.
Feedback: INCORRECT! In the case of a conventional pledge where the proceeds are less
than the amount of debt, the creditor shall not be entitled to recover the deficiency,
notwithstanding any stipulation to the contrary.
Score: 0 out of 1 No
Question 7
A sold to B a residential house and lot at an installment price of P6,000,000 payable in 50
years at P10,000 per month. After payment of total installments amounting to P240,000, B
defaulted on the next installment. After following the necessary procedures required by law, the
contract of sale is properly canceled. How much cash surrender value is B entitled from A?
Response: P120,000
Feedback:
CORRECT! Since the sale is canceled, the seller shall refund to the buyer the cash surrender
value of the payments on the property equivalent to 50% of the total payments made.

(P240,000/P10,000 = 24 months or 2 years)

P240,000 x 50% = P120,000


Score: 1 out of 1 Yes
Question 8
On January 1, John sold a registered land to Judy in a notarized document. On January 2,
John sold the same land again to Jude under a notarized deed of sale. Jude registered the
deed of sale to the Registry of Property on the same date. On January 3, John sold the same
land again to Jed under a private deed of sale. Jed took physical possession of the land on the
same date. Who has a better right over the land, assuming all buyers do not know about the
sale to one another?
Response: Jude
Feedback: CORRECT! Jude has a better right over the land because he is the first registrant in
good faith.
Score: 1 out of 1 Yes
Question 9
M owes P10,000 to P, evidenced by a promissory note. Subsequently, P assigned the note to
A, A to B, B to C, and C back to M. In this case, how is the obligation of M extinguished?
Response: By confusion
Feedback: CORRECT! The obligation is extinguished by meeting in one person the qualities or
characters of the creditor and debtor, i.e., M.
Score: 1 out of 1 Yes
Question 10
Christian borrowed P10,000 from James. Christian pledged 50 grams of methamphetamine
(shabu). On the due date, James demanded payment, but Christian refused to pay by raising
the defense that his debt is void. May James collect from Christian?
Response: Yes, but if Christian cannot pay, James may resort to other remedies to satisfy his
claim.
Feedback: CORRECT! James can collect from Christian, but if Christian cannot pay, James
may have to resort to other remedies than going after the pledge because the pledge is void.

LAWS ON BUSINESS ORGANIZATION I


Question 1
When will a merger or consolidation of cooperatives be valid?
Response: Approval by three-fourths (¾) vote of all the members with voting rights
Feedback: CORRECT! The affirmative vote of members representing at least three-fourths
(3/4) of all members with voting rights, present and constituting a quorum of each of the
constituent cooperatives at separate general/representative assembly meetings shall be
necessary for the approval of the Plan of Merger or Consolidation, or amendments to it if any.
Score: 1 out of 1 Yes
Question 2
Which of the following stipulations in sharing profit or losses is valid?
Response: Excluding an industrial partner in the share of profits.
Feedback: INCORRECT! Article 1799 of the New Civil Code provides a stipulation that
excludes one (1) or more partners from any share in the profits or losses is void.
Score: 0 out of 1 No
Question 3
Lorreyn and Patrick entered a partnership contract for a fixed term of two (2) years. Lorreyn is a
capitalist partner, while Patrick is an industrial partner. They agreed that Patrick would share
equally in the profit. No agreement was made as to losses.
On 20X1, the partnership had P10,000 net loss. On 20X2, the partnership had P20,000 net
income. Upon examining the contract, it was determined that the partners intended the word
profit as cumulative profit.
How much Patrick will receive for his share in the two (2) years of partnership operation?
Response: P0 because an industrial partner has no investment
Feedback: INCORRECT! Article 1797 of the New Civil Code provides that an industrial partner
may receive profit as may be just and equitable under the circumstances.
Score: 0 out of 1 No
Question 4
Which of the following cooperative principles shows the principle of “voluntary and open
membership”?
Response: Impartiality
Feedback: CORRECT! This principle states that cooperatives are voluntary organizations,
open to all persons able to use their services and willing to accept the responsibilities of
membership, without gender, social, racial, cultural, political, or religious discrimination.
Score: 1 out of 1 Yes
Question 5
An insane and capacitated person orally entered into a contract of partnership. The insane
contributed a car while the capacitated person contributed P1 Million cash. What is the status
of the contract of partnership?
Response: Voidable on the part of the insane partner.
Feedback: CORRECT! The insane person is incapable of giving consent; thus, the contract of
partnership is voidable on his/her part.
Score: 1 out of 1 Yes
Question 6
What is the required vote for the amendment of the provisions of By-Laws of a Cooperative?
Response: At least 2/3 of all members with voting rights
Feedback: CORRECT! This is provided in Article 18 of Republic Act 9520.
Score: 1 out of 1 Yes
Question 7
Which of the following statements is TRUE about the rights of a capitalist partner to engage in
another business?
Response: A capitalist partner can engage in a business of different kinds even without
stipulation allowing him to do so and in a business of the same kind if a stipulation allows him
to do so.
Feedback:
CORRECT! Article 1808 of the New Civil Code prohibits capitalist partners from engaging in
business of the same kind with the partnership unless there is a stipulation to the contrary. Any
capitalist partner violating the prohibition shall bring any profits accruing to the common funds
from his transactions and shall personally bear all the losses.
Score: 1 out of 1 Yes
Question 8
There are nine (9) seats on a cooperative’s Board of Directors. Five (5) seats are vacant due
to death, resignation, disqualification, and removal of a director. How shall the five (5)
vacancies be filled up?
Response: By the votes of the general assembly
Feedback: CORRECT! Article 41 of the amended Philippine Cooperative Act provides that any
vacancy in the board of directors, other than by expiration of term, may be filled by the vote of
at least a majority of the remaining directors if still constituting a quorum. Otherwise, the
general assembly must fill the vacancy in a regular or special meeting called for the purpose.
Score: 1 out of 1 Yes
Question 9
What is the minimum paid-up capital before a prospective primary cooperative is registered
with the Cooperative Development Authority (CDA)?
Response: At least 25% of the actual total subscription or P15,000, whichever is higher.
Feedback: CORRECT! Section 2 of Article VIII of MC 2015-01, otherwise known as the
Revised Guidelines Governing the Registration of Cooperatives, provides the capital
requirements of primary cooperatives.
Score: 1 out of 1 Yes
Question 10
Which of the following is generally NOT allowed to enter into a partnership contract?
Response: A corporation
Feedback: CORRECT! As a general rule, a corporation cannot become a partner. This
limitation is based on public policy. In a partnership, the corporation would be bound by the
acts of persons who are not duly appointed and authorized agents and officers. This would be
entirely inconsistent with the policy of the law that the corporation shall manage its affairs
separately and exclusively.

LAW ON BUSINESS ORGANIZATION II


Question 1
What is the required vote for the sale or disposal by SM Group of SM Mall of Asia?
Response: Approval by at least a majority vote of the present members of directors in a
meeting where there is a quorum.
Feedback: CORRECT! This is provided in Section 39 of the Revised Corporation Code.
Score: 1 out of 1 Yes
Question 2
It refers to the action brought by a stockholder against the corporation for direct violation of his
contractual rights.
Response: Individual Suit
Feedback: CORRECT! An example of an application of an individual suit is when the
stockholder is denied his right to inspect the corporate records or his right to dividend.
Score: 1 out of 1 Yes
Question 3
HONDA Philippines is fully owned by Japanese nationals. Its main office is located in Tokyo,
Japan, but it was incorporated under the Corporation Code of the Philippines. It engages its
business in the Philippines. What is the classification of the corporation under the Corporation
Code?
Response: Domestic corporation
Feedback: CORRECT! This is a corporation formed, organized, or existing under Philippine
laws.
Score: 1 out of 1 Yes
Question 4
Which of the following dividend declarations must be ratified by the stockholders owning at
least 2/3 of the outstanding capital stock entitled to vote?
Response: Stock dividend
Feedback: CORRECT! This is provided in Article 42 of the Revised Corporation Code.
Score: 1 out of 1 Yes
Question 5
What is the required vote in increasing/decreasing the authorized capital stock?
Response: Approval by at least a majority vote of the board of directors and ratification by
stockholders representing at least 2/3 of the outstanding capital stock.
Feedback: CORRECT! This is provided in Section 37 of the Revised Corporation Code.
Score: 1 out of 1 Yes
Question 6
What is the required vote for the declaration of cash or property dividend?
Response: Approval by at least a majority vote of the board of directors.
Feedback:
CORRECT! This is applicable in the following cases, among others:
 Declaration of cash or property dividend; and
 Acquisition of treasury shares.
Score: 1 out of 1 Yes
Question 7
What formality is required by the Revised Corporation Code for the validity and enforceability
of a subscription contract?
Response: It may be in any form because it is perfected by mere consent.
Feedback: CORRECT! Mere consent is the requirement for a contract to be perfected.
Score: 1 out of 1 Yes
Question 8
A director, who by his office, acquired for himself a business opportunity which should belong
to the corporation, thereby obtaining profits to the prejudice of the corporation. Is the director
liable?
Response: No, the director is not liable if he did not act in bad faith.
Feedback: INCORRECT! Bad faith is not a requisite for the director to be liable.
Score: 0 out of 1 No
Question 9
Which of the following determines the state of incorporation, regardless of the nationality of the
stockholders?
Response: Control test
Feedback: INCORRECT. This determines the nationality of the controlling stockholders or
members and is used to identify nationality for investment purposes.
Score: 0 out of 1 No
Question 10
Which of the following is a ground for involuntary dissolution of a corporation?
Response: By non-use of corporate charter
Feedback: CORRECT! This is under Section 21 of the Corporation Code, which states that if a
corporation does not formally organize and commence its business within five (5) years from
the date of its incorporation, its certificate of incorporation shall be deemed revoked as of the
day following the end of the five (5)-year period.
SPECIAL LAWS I
Question 1
Which of the following petition may be filed by an individual debtor who does not have
sufficient funds and wants to be discharged from his liabilities that exceeds P500,000?
Response: Petition for voluntary liquidation of insolvent individual debtor
Feedback: CORRECT! The individual debtor files the petition for voluntary liquidation, but the
debtor is insolvent or has no sufficient assets to cover the liabilities.
Score: 1 out of 1 Yes
Question 2
Brix Tejano has total assets worth P1,500,000 and total liabilities of P4,000,000. Under
the Financial Rehabilitation and Insolvency Act (FRIA), what legal remedy shall Brix avail to be
discharged from his liabilities?
Response: Petition for involuntary liquidation
Feedback: INCORRECT! This legal remedy is available for the creditors.
Score: 0 out of 1 No
Question 3
What is the required approval in filing a petition for voluntary rehabilitation under the Financial
Rehabilitation and Insolvency Act (FRIA)?
Response: A majority vote of the Board of Directors and authorized by the vote of the
stockholders representing at least 2/3s of the outstanding capital stock.
Feedback: CORRECT! This is the required vote provided in Section 12 of FRIA. Moreover, this
is the ONLY provision in FRIA wherein the directors and stockholders will vote. All other voting
involves the participation of debtors and creditors.
Score: 1 out of 1 Yes
Question 4
Which of the following is NOT an element of a violation of Batas Pambansa (BP) 22, otherwise
known as the Bouncing Checks Law, for issuing a worthless check or no sufficient fund (NSF)
check?
Response: There must be knowledge of the maker, drawer, or issuer that at the time of issue,
he does not have sufficient funds in or credit with the drawee bank for the payment of the
check, in full, upon its presentment.
Feedback: INCORRECT! This is one of the elements of the violation of BP 22 as provided in
Section 1 of the Act.
Score: 0 out of 1 No
Question 5
This is the unpaid balance of money or its equivalent received by a bank in the usual course of
business.
Response: Deposit
Feedback: CORRECT! These are money or funds placed with a bank that can be withdrawn on
the depositor’s order or demand, such as deposit accounts in savings and current and time
deposits. Deposits are characterized as being like a simple loan.
Score: 1 out of 1 Yes
Question 6
Which of the following coins or currencies may still be replaced by Bangko Sentral ng Pilipinas
(BSP) with compensation to the bearer?
Response: Notes and coins with drawings
Feedback: CORRECT! Notes and coins with drawings can still be replaced with compensation
to the bearer because it is still identifiable.
Score: 1 out of 1 Yes
Question 7
What is the quantum of evidence in awarding civil damages for the issuance of worthless
checks?
Response: Preponderance of evidence
Feedback: CORRECT! This is the required quantum of evidence for civil cases.
Score: 1 out of 1 Yes
Question 8
Allan has total assets worth P2,000,000 and total liabilities amounting to P1,300,000.
However, most of his assets are fixed assets that cannot be easily sold, while most of his
liabilities will mature within one (1) month. What legal remedy shall Timothy avail of under the
Financial Rehabilitation and Insolvency Act (FRIA)?
Response: Petition for declaration of suspension of payments
Feedback: CORRECT! The individual debtor shall file a petition for suspension of payments
because he has enough assets to cover liabilities.
Score: 1 out of 1 Yes
Question 9
What is the quantum of evidence required for conviction of violating the Batas Pambansa (BP)
22, otherwise known as the Bouncing Check Law, for issuing worthless checks?
Response: Proof beyond reasonable doubt
Feedback: CORRECT! This is the required quantum of evidence for criminal cases.
Score: 1 out of 1 Yes
Question 10
Which of the following statements concerning the maximum legal tender in the Philippines
is INCORRECT?
Response: For 1-centavo, 5-centavo, and 25-centavo coins - up to P1,000 maximum legal
tender
Feedback: CORRECT! According to BSP Circular No. 537 Series of 2006, 1-centavo, 5-
centavo, 10-centavo, and 25-centavo coins are legal tender for a maximum of one hundred
pesos (P100.00).

SPECIAL LAWS PART 2


Question 1
Which is NOT a right of the data subject?
Response: Right to inspect the personal information of other data subject
Feedback: CORRECT! Just as the data subject wants their information to be protected, so do
others. The data subject has no right to inspect the personal information of other data subjects.
Score: 1 out of 1 Yes
Question 2
Daniel has the following deposit accounts in a closed bank: Daniel, P350,000; Daniel and
Ezekiel, P600,000. What is the maximum deposit insurance coverage insofar as Daniel is
concerned?
Response: P600,000
Feedback: CORRECT! For the single account, Daniel is entitled to P350,000. For the joint
account, he is entitled only to P250,000, for a total of P600,000. Of the P600,000 in the joint
account, only P500,000 is insured by the Philippine Deposit Insurance Corporation (PDIC).
This means that each of the depositors is entitled only to P250,000.
Score: 1 out of 1 Yes
Question 3
Which intellectual property is created or protected from the moment of registration with the
Intellectual Property Office (IPO
Response: Patent and trademark
Feedback: CORRECT! Patents and trademarks are protected from the moment of registration
with the Intellectual Property Office (IPO).
Score: 1 out of 1 Yes
Question 4
As a general rule, what is the quantitative threshold for any transaction in cash or other
equivalents of a covered person to be considered a reportable transaction to the Anti-Money
Laundering Council (AMLC)?
Response: A transaction with an amount exceeding P500,000
Feedback: CORRECT! A transaction in cash or other equivalent monetary instrument
exceeding P500,000 is a covered transaction under the Anti-Money Laundering Act (AMLA).
Score: 1 out of 1 Yes
Question 5
Under the Republic Act No. 1405, which of the following is NOT considered absolutely
confidential in nature?
Response: Money market placement
Feedback: CORRECT! Money market placement is not a kind of deposit.
Score: 1 out of 1 Yes
Question 6
Which of the following is NOT a predicate crime of money laundering offense?
Response: Adultery
Feedback: CORRECT! This is not a predicate crime under the Anti-Money Laundering Act
(AMLA) because it does not involve money laundering
Score: 1 out of 1 Yes
Question 7
Which of the following is NOT a sensitive personal information?
Response: Which of the following is NOT a sensitive personal information?
Feedback: CORRECT! This is a personal information but not considered sensitive personal
information.
Score: 1 out of 1 Yes
Question 8
Isaiah has the following deposit accounts in a closed bank: Isaiah, P150,000, and Isaiah’s
retail business (sole proprietorship), P400,000. What is the maximum deposit insurance
coverage insofar as Isaiah is concerned?
Response: P500,000
Feedback: CORRECT! Even if Isaiah has P550,000 in his deposit account, the maximum
deposit insurance coverage is P500,000 only. The excess of P50,000 is not covered.
Score: 1 out of 1 Yes
Question 9
Which of the following deposits is NOT insured by the Philippine Deposit Insurance
Corporation (PDIC)?
Response: Savings deposit in Cainta UnionBank
Feedback: INCORRECT! Membership with PDIC is mandatory for all banks licensed by the
BSP to operate in the Philippines; therefore, savings deposit in Cainta UnionBank is insured by
PDIC.
Score: 0 out of 1 No
Question 10
Lorrenz Pinon is an employed reporter of ABS-GMA Co. During his free time, he was able to
write a novel, True Love, using the laptop and printer of the company. Who owns the copyright
over the novel?
Response: Lorrenz Pinon
Feedback: CORRECT! In the case of original literary and artistic works, copyright shall belong
to the author of the work. Also, in the case of work for hires, if the creation of the object of
copyright is not a part of his regular duties, even if the employee uses the employer's time,
facilities, and materials, the copyright belongs to the employee.

TAXATION
INCOME TAX - INDIVIDUALS

Question 1
In 2022, Ysa Batumbakal, the Assurance Manager of Deloitte Philippines, earned a
compensation income of P2,001,000 in the Philippines. This amount is inclusive of his 13th-
month pay and other benefits of P800,000, but net of mandatory contributions to SSS and
PhilHealth.
She also owns a restaurant in Metro Manila. In 2022, the restaurant has gross receipts of
P2,000,000, cost of sales of P500,000, and operating expenses of P600,000. It also had a non-
operating income of P235,000.
What would be Ysa’s total income tax due if she is a non-resident citizen and avails of the 8%
optional income tax rate?
Response: P642, 100
Feedback:
Correct! If a mixed earner signifies his intention to be taxed at an 8% optional income tax rate,
his income from compensation will be subjected to graduated rates. In comparison, his income
on self-employment or practice of profession will be subjected to the 8% optional income tax
rate.
Thus, the tax for his compensation income:

Total Compensation Income, Net of 13th-


month benefits P1,911,000
(P2,001,000 – P90,000)

Tax on the First P800,000 P130,000


Tax on the Excess
333,300
(P 1,911,000 – P800,000) x 30%
Tax Due on Compensation 463,300
The 8% on total gross receipts:

Gross Receipts P2,000,000


Add: Other Income 235,000
Total Income Subject to 8% P2,235,000
Multiply by: 8%
Tax Due on Business P178,800
Getting the total of the two (2) tax dues:

Tax Due on Compensation P463,300


Tax Due on Business 178,800
Total Income Tax Due P642, 100

Score: 1 out of 1 Yes


Question 2
In 2022, Chester Acob, Senior Manager of Ernst & Young Philippines, earned a compensation
income of P4,000,500. This amount is inclusive of his 13th-month pay and other benefits of
P250,000, but net of mandatory contributions to SSS and PhilHealth.
Aside from his employment, he also operates a shoe store. In 2022, the shoe store has a
gross sale of P 1,500,000, cost of sales of P500,000, and operating expenses of P300,000. It
also had a non-operating income of P700,000.
What would be Chester’s total income tax due if he is a resident citizen and he did not avail of
the 8% optional income tax rate?
Response: P1,549,360
Feedback:
Correct! If a mixed earner failed to signify his intention to be taxed at the 8% optional income
tax rate, he would be taxed at graduated rates plus a 3% percentage tax.
First, get the total income:

Income from Compensation


P3,910,500
(P4,000,500 – P90,00)

Gross Receipts P1,500,000


Add: Other Income 700,000
Total Income P2,200,000
Less: Cost of Sales 500,000
Operating Expenses 300,000
Net Income from Business P1,400,000

Income from Compensation P1,500,000


Add: Income from Business 700,000
Total Income P5,310,500
Then, compute the tax due using the income tax table:

Tax on the First P2,000,000 P490,000


Add: Tax on the Excess
1,059,360
(P5,310,500 – P2,000,000) x 32%
Total Income Tax Due P1,549,360
Score: 1 out of 1 Yes
Question 3
ABC Corporation, a Philippine corporation, has an executive (Juan) who is a Filipino citizen.
ABC has a subsidiary in Singapore (Singapore Corporation) and will assign Juan for an
indefinite period to work full-time for Singapore Corporation. Juan will bring his family to
Singapore and lease his residence in the Philippines. Juan's salary will be shouldered 50% by
ABC Corp. while the other 50% and housing, cost of living, and educational allowances of
Juan's dependents will be shouldered by Singapore Corporation. ABC Corp. will credit 50% of
Juan's salary to Juan's Philippines bank account. Juan will sign the contract of employment in
the Philippines. Juan will also receive rental income for the lease of his Philippine residence.
Which of the following income will be subject to Philippine income tax?
Response: Rental income
Feedback: CORRECT! The rental income for the lease of Juan's residence is derived from
"within" the Philippines, the leased property in the Philippines. Hence, taxable to Juan.
Score: 1 out of 1 Yes
Question 4
Angeline Quizon, a resident citizen, won the following prizes during the year:

First
Singing contest he voluntarily joined P10,000
Prize
First Philippine Charity Sweepstakes Office
500,000
Prize (PCSO) winnings
Third
Raffle ticket winnings 8,000
Prize

Which of the following is TRUE under TRAIN Law?


Response: The winnings from the raffle ticket are subject to a 20% final tax.
Feedback: Correct! The winnings from the raffle ticket were earned by chance, hence subject
to a 20% final tax.
Score: 1 out of 1 Yes
Question 5
What will be your possible action when the Court of Tax Appeal (CTA) denies your appeal for
protest?
Response: File a motion for reconsideration to the CTA division within 15 days from the receipt
of the decision
Feedback: Correct! After you file for a protest and the CTA denies it, your possible action will
be filing for a motion for reconsideration. You cannot just go directly to the CTA en banc, the
SC, and SC en banc.
Score: 1 out of 1 Yes
Question 6
Five (5) years ago, Sunshine Santos bought 5,000 shares of TXT Corporation (domestic, not
listed) at par value. She sold her shares for P10 million today. The corporation has 10,000
outstanding shares with a par value of P1,000/share. Per its latest Financial Statements, the
corporation’s assets totaled P30,000,000, and its liabilities totaled P5 million. The book values
of TXT’s assets and liabilities are equivalent to their market values except for its real property.
The book, market, zonal, and appraised values of TXT’s real properties are as follows:
Book Value MV per Tax Appraised
Zonal Value
per FS Declaration Value
Land A P2,000,000 P2,500,000 P5,000,000 P6,000,000
Land B 2,000,000 2,200,000 4,000,000 3,500,000
Building
1,000,000 2,400,000 3,000,000
A
Building
500,0000 2,000,000 1,950,000
B
How much is the capital gains tax due on the sale of Sunshine’s shares of stock?
Response: P750,000
Feedback:
CORRECT! This can be computed as follows:

Selling Price P10,000,000


Less: Cost Basis (5,000,000)
Gain P5,000,000
Rate 15%
Capital Gains Tax P750,000

Score: 1 out of 1 Yes


Question 7
A taxpayer received during the taxable year the following passive income derived from within
the Philippines:
Interest on bank deposits under FCDU (net) P231,250
Royalty on a software application (gross) 95,000
Dividend income RFC (gross) 150,000
If the taxpayer is a non-resident alien engaged in business, the final tax on the above passive
income will amount to:
Response: P19,000
Feedback:
CORRECT! The final withholding tax (FWT) on royalty income is computed as follows:
Royalty on a software application (gross) P95,000
Multiply: Rate 20%
P19,000
Score: 1 out of 1 Yes
Question 8
Upon retirement, Sandara Dimagiba plans to sell a parcel of idle land (capital asset)
in Calamba, Laguna, which she inherited from her father 20 years ago. Before selling,
she discovered that other parcels of land of the same size and in the same area sold for P2.8
million. She even hired a professional appraiser who estimated the value of her land at P3.0
million. She eventually sold the parcel of idle land to her friend for P2.4 million. At the time of
sale, the zonal value was P2.6 million, and the assessor’s value was P2.7 million.
How much is the capital gains tax (CGT) on the sale?
Response: P162,000
Feedback:
CORRECT! Section 24, D(1) of the Amended Tax Code provides that a 6% capital gains tax
shall be imposed on the selling price or current fair market value of the sold real property.
The current fair market value of the real property is equal to the higher between the fair market
value as determined by the Commissioner and the values shown by the Provincial and City
assessors.
Thus, the capital gains tax can be computed in the given case by multiplying P2.7 million with
6%.
Score: 1 out of 1 Yes
Question 9
Carl John Garcia is a minimum wage earner (MWE) working in Manggahan, Pasig. As such,
his employer did not withhold any tax from his salary. Starting April 2022, he was given a raise
in salary, which was more than the statutory minimum wage. Will his entire compensation for
2022 be taxable and subject to withholding tax?
Response: Only his earnings from April 2022 to December 2022 shall be taxable and subject to
withholding tax
Feedback: Correct! Only those portions of Carl John’s compensation income will be taxable
and subject to withholding tax, which is by the time his salary exceeds the minimum wage rate.
Score: 1 out of 1 Yes
Question 10
On January 1, 2022, Rachelle Perez invested P1,000,000 in a 12% long-term deposit
certificate (BSP-prescribed form) with a five-year maturity. She plans to pre-terminate the said
investment on December 31, 2024. What is the final tax imposed on the entire income on the
said pre-termination plan?
Response: 20%
Feedback: Correct! This final tax will be imposed if the remaining maturity from the pre-
termination is less than three (3) years.

INCOME TAXATION - CORPORATION


Question 1
The record of Segunda Corporation (SC), a closely held corporation, shows the following data
for 2022:

Gross Income P1,300,000


Business Expenses 600,00
Gain on sale of business asset 50,000
Interest on deposits with Metrobank, net of tax 5,000
Dividends from Paul, a domestic corporation 35,000
Dividend paid during the year 130,000
Reserved for plant acquisition 400, 000
In 2021, the corporation suffered an operating loss of P150,000. This amount was carried
forward and claimed as a deduction from gross income in 2022. What is SC's improperly
accumulated earnings tax (IAET) for 2022 following the Corporate Recovery and Tax
Incentives for Enterprises (CREATE) Act?
Response: P-0-
Feedback: CORRECT! The 10% IAET has been repealed by the CREATE Act. In Revenue
Regulations (RR) No. 5-2021, the Bureau of Internal Revenue (BIR) provides that the repeal of
IAET applies to the entire taxable year for all fiscal years/taxable years ending after the
effectivity of CREATE.
Score: 1 out of 1 Yes
Question 2
Where should he file his return if the person has no legal residence or place of business in the
Philippines?
Response: Office of the Commissioner
Feedback: Correct! The office of the Commissioner is located at RDO No. 39, South Quezon
City.
Score: 1 out of 1 Yes
Question 3
Which of the following DOES NOT fall under the definition of a “corporation” for income tax
purposes according to Section 22(B) of the National Internal Revenue Code (NIRC)?
Response: Joint-stock company
Feedback: Correct! A resident foreign engaged in trade or business in the Philippines through a
branch is subject to 10% corporate income tax and hence can be subjected to MCIT.
Score: 1 out of 1 Yes
Question 4
Assuming today is May 2023, which of the following is INCORRECT about the Corporate
Recovery and Tax Incentives for Enterprises (CREATE) Act?
Response: Non-profit and proprietary educational institutions and hospitals are subject to a
special corporate income tax of 10%.
Feedback: CORRECT! This is the incorrect statement. Under the CREATE Act, non-profit and
proprietary educational institutions and hospitals are subject to a 1% special corporate income
tax.
Score: 1 out of 1 Yes
Question 5
Anna Corporation (domestic), which started operations in 2010, has the following data for the
fiscal year ending April 30, 2021:
Sales, net of sales discounts P20,000,000
Cost of sales 5,000,000
Salaries of employees, net of payroll deductions
5,000,000
of P350,000
Fringe benefits given to the rank and file
1,040,000
employees
Fringe benefits given to the managerial
325,000
employees
Entertainment, Amusement, and
550,0000
Representation (EAR) expenses
Rent expenses 1,200,000
Depreciation expense 700,000
Bad debt expense (1/3 charged off during the
105,000
year)
Interest expense on BPI loan) 400,000
Interest expense on loan from majority
100,000
shareholder

Other income:
 Cash dividends received from:
 Domestic corporations, P550,000
 Foreign corporations, P30,000
 Interest income from Philippine bank deposits, net of final tax, P100,000
 Royalty income in the Philippines, gross of final tax, P125,000
 Gain from sale of real property:
 Makati real property not used in business (Selling Price = P10 million), P2,000,000
 Domestic shares (not listed) held as capital assets, P100,000
 Domestic shares (listed) held as capital assets, P23,000,000
 Liquidating dividend from Kyu Corp. (cost of Kyu Corp.’s shares = P96,000), P100,000
 Creditable withholding tax (CWT) withheld by customers, P68,000
 Tax paid in first three (3) quarters, P30,000

How much is the corporation’s regular corporate income tax (RCIT) due?
Response: P1,481,793
Feedback:
CORRECT! First, solve for the blended rates for regular corporate income tax (RCIT), minimum
corporate income tax (MCIT), and interest rate arbitrage:

Months RCIT MCIT Arbitrage


May 1,2020 to
2 30% 2% 33%
June 30, 2020
July 1, 2020, to
10 25% 1% 20%
April 30, 2021

(30% x (2/12) (2% x (2/12)


(33% x2/10) +
+
+ (25%x(10/12) (1%x(10/12) (20% x
= 25.83% (10/12)= 22.17%
=1.17%

Afterward, solve for the net taxable income:

Net Sales P20,000,000


Cost of sales (5,000,000)
Gross income from operations 15,000,000
Other taxable income 30,000
Net capital gains – liquidating dividend 4,000 34,000
Total gross income 15,034,000
Less itemized deductions:
Salary expense 5,000,000
Payroll deductions 350,000 (5,350,000)
Fringe benefits
Rank and file 1,040,000
Managerial 325,000 (1,365,000)
Fringe benefit tax (175,000)
Entertainment and representation –
(100,000)
limit
Rent expense (1,200,000)
Bad debt expense (1/3 charged off) (700,000)
Deductible interest expense 400,000
Interest arbitrage (P12,500 x 22.17%) (27,713) (372,287)
Net taxable income P5,736,713

Then solve for the RCIT:

Net taxable income P5,736,713


Regular corporate income tax rate
25.83%
(blended)
Income tax due P1,481,793

Score: 1 out of 1 Yes


Question 6
John Corporation (JC) has the following data for 2020:

Excess MCIT 2019 P10,000


Quarter 1st 2nd
Income per quarter P500,000 P800,000
Deductions 480, 000 700, 000
JC’s taxable income and total assets for 2020 will not exceed P5 million and P100 million,
respectively.
How much is JC’s income tax payable in the second quarterly return?
Response: P16,000
Feedback:
Correct! Determine the tax due to the company under the 30% Regular Income Tax (RIT) first:

Quarter 1st 2nd


Cumulative Gross Income P500,000 P1,300,00
Less: Cumulative Deductions 480,000 1,180,00
Taxable Net Income P20,000 P120,000
Multiply by: 30% 30%
Income Tax Due P6,000 P36,000

Then, determine the tax due under 2% Minimum Corporate Income Tax (MCIT):

Quarter 1st 2nd


Cumulative Gross Income P500,000 P1,300,00
Multiply by: 2% 2%
Income Tax Due P10,000 P26,000

By comparing the tax due under each tax, it is determined that in the 1st quarter, the company
will pay the tax due under MCIT. Thus, the prior year’s MCIT cannot be credited:

Quarter 1st 2nd


Income Tax Due P10,000 P36,000
Less: MCIT from Prior Year 10,000
Income Tax Paid 10,000
Income Tax Payable P10,000 P16,000

Score: 1 out of 1 Yes


Question 7
A branch profit remittance is subject to what percentage of tax?
Response: 15%
Feedback: Correct! This is a tax to be paid aside from the income tax of 30%.
Score: 1 out of 1 Yes
Question 8
Yes Corporation (YC) has the following data for 2022:

Excess MCIT 2021 P10,000


Quarter 1st 2nd
Income per quarter P500,000 P800,000
Deductions 480, 000 700, 000
YC’s taxable income and total assets for 2022 will not exceed P5 million and P100 million,
respectively.
Under the Comprehensive Recovery and Tax Incentives for Enterprises (CREATE) Act, how
much is YC’s income tax payable in the second quarterly return?
Response: P9,000
Feedback:
Correct! Determine the tax due to the company under the 20% Regular Income Tax (RIT) first:

Quarter 1st 2nd


Cumulative Gross Income P500,000 P1,300,00
Less: Cumulative Deductions P480,000 P1,180,00
Taxable Net Income P20,000 P120,000
Multiply by: 20% 20%
Income Tax Due P4,000 P24,000

Then, determine the tax due under 1% Minimum Corporate Income Tax (MCIT):

Quarter 1st 2nd


Cumulative Gross Income P500,000 P1,300,000
Multiply by: 1% 1%
Income Tax Due P5,000 P13,000

By comparing the tax due under each tax, it is determined that in the 1st quarter, the company
will pay the tax due under MCIT. Thus, the prior year’s MCIT cannot be credited:

Quarter 1st 2nd


Income Tax Due P5,000 P24,000
Less: MCIT from Prior Year 10,000
Income Tax Paid 5,000
Income Tax Payable P5,000 P9,000

Score: 1 out of 1 Yes


Question 9
The Minimum Corporate Income Tax (MCIT) shall not apply to the following resident foreign
corporations, EXCEPT:
Response: A resident foreign corporation engaged in trade or business in the Philippines
through a branch office
Feedback: Correct! A resident foreign engaged in trade or business in the Philippines through a
branch is subject to 10% corporate income tax and hence can be subjected to MCIT.
Score: 1 out of 1 Yes
Question 10
Anna Corporation (domestic), which started operations in 2010, has the following data for the
fiscal year ending April 30, 2021:
Sales, net of sales discounts P20,000,000
Cost of sales 5,000,000
Salaries of employees, net of payroll deductions of
5,000,000
P350,000
Fringe benefits given to rank and file employees 1,040,000
Fringe benefits given to managerial employees 325,000
Entertainment, Amusement, and Representation
550,0000
(EAR) expenses
Rent expenses 1,200,000
Depreciation expense 700,000
Bad debt expense (1/3 charged off during the year) 105,000
Interest expense on BPI loan) 400,000
Interest expense on loan from majority shareholder 100,000

Other income:
 Cash dividends received from:
 Domestic corporations, P550,000
 Foreign corporations, P30,000
 Interest income from Philippine bank deposits, net of final tax, P100,000
 Royalty income in the Philippines, gross of final tax, P125,000
 Gain from sale of real property:
 Makati real property not used in business (Selling Price = P10 million), P2,000,000
 Domestic shares (not listed) held as capital assets, P100,000
 Domestic shares (listed) held as capital assets, P23,000,000
 Liquidating dividend from Kyu Corp. (cost of Kyu Corp.’s shares = P96,000), P100,000
 Creditable withholding tax (CWT) withheld by customers, P68,000
 Tax paid in first three (3) quarters, P30,000

How much is the corporation’s capital gains tax (CGT) applying the provision of the Corporate
Recovery and Tax Incentives for Enterprises (CREATE) Act?
Response: P615,000
Feedback:
CORRECT! This can be computed as follows:

Capital gains tax on Makati property (P10,000,000 x 6%) P600,000


Domestic shares as capital assets (P100,000 x 15%) 15,000
Capital gains tax P615,000
Score: 1 out of 1
ESTATE AND DONOR’S TAX
Question 1
Mr. Jack Batumbakal revocably transfers his penthouse to his nephew. He challenges his
nephew to pass the Civil Engineering Licensure Examination because if he does, he will waive
his power to revoke the transfer. The nephew tops the examination. Mr. Batumbakal waives his
power to revoke the transfer. The transfer is subject to:
Response: Donor’s tax
Feedback: Correct! Upon the waiving of Mr. Batumbakal on his right to revoke the transfer, the
said transaction will now be subjected to the donor’s tax.
Score: 1 out of 1 Yes
Question 2
Mr. Dimagiba died, leaving the following properties and estate deductions:

Family Home P11,050,000


Separate properties of Mr. Dimagiba 8,000,000
Communal Properties 4,700,000
Ordinary deductions of communal
800,000
properties
Ordinary deductions - exclusive of Mr.
400,000
Dimagiba
Ordinary deductions - exclusive of Mrs.
500,000
Dimagiba

What is the estate tax due?


Response: P4,500
Feedback:
Correct! This can be computed as follows:

Separate Common Total


Family Home P11,050,000 P11,050,000
Communal Properties 4,700,000 4,700,000
Separate Property 8,000,000 8,000,000
Less: Ordinary Deductions 400,000 800,000 1,200,000
Net Estate Before Special P7,600,00
P14,950,000 P22,550,000
Deductions 0
Less: Special Deductions
Family Home 10,000,000
Standard Deductions 5,000,000
Net Estate before surviving spouse P7,550,000
Share of the surviving spouse 7,475,000
Taxable Net Estate P75,000
Multiply by: 6%
Estate Tax Due P4,500
Score: 1 out of 1 Yes
Question 3
Mr. Choi, a resident citizen, was single when he died. He left properties he inherited 3 ½ years
ago with a current fair market value of P950,000. The property was inherited when it was worth
P1,350,000 and had a P800,000 unpaid mortgage. Mr. Choi paid P500,000 until his death.
Other properties of Mr. Choi had a fair market value of P3,050,000 at the time of his death. The
value of losses, taxes, and transfers for a public purpose is P150,000. How much is the
vanishing deduction? (Do not round off your answers.)
Response: P142,000
Feedback:
Correct! First, determine the property's fair market value, which is equal to the lower between
the fair market value (FMV) at the date of inheritance or donation from the prior decedent and
the FMV at the time of the present decedent.

Initial value (lower of P800,000 and


P1,350,000
P1,200,000)
Less: Mortgage assumed and paid 950,000
Initial Basis P400,000
Less: Proportional other ordinary deductions
Initial basis / Gross estate x Other ordinary
deductions (P400,000 / P4,000,000) x 45,000
(P450,000)
Final Basis P355,000
Multiplied by: Vanishing Percentage (3 years) 40%
Vanishing Deduction P142,000
Score: 1 out of 1 Yes
Question 4
Five (5) years ago, Veronica bought 5,000 shares of TST Corporation (domestic, not listed) at
par value. She sold her shares for P10 million today. The corporation has 10,000 outstanding
shares with a par value of P1,000/share. Per its latest Financial Statements, the corporation’s
assets totaled P30,000,000, and its liabilities totaled P5 million. The book values of TST’s
assets and liabilities are equivalent to their market values except for its real property.

The book, market, zonal, and appraised values of TST’s real properties are as follows:
MV per
Book Value Zonal Appraised
Tax
per FS Value Value
Declaration
P2,000,00 P2,500,00 P5,000,00
Land A P6,000,000
0 0 0
Land B 2,000,000 2,200,000 4,000,000 3,500,000
Building A 1,000,000 2,400,000 3,000,000
Building B 500,0000 2,000,000 1,950,000

How much is the donor’s tax due, if any?


Response: P90,000
Feedback: Incorrect! In the case of shares of stocks donation, the fair market value shall
depend on whether the shares are listed or unlisted on the stock exchanges. Unlisted common
shares are valued based on their book value, while unlisted preferred shares are valued at par
value.
Score: 0 out of 1 No
Question 5
All of the following are allowed to be deducted from the gross gift under Tax Reform
Acceleration and Inclusion (TRAIN) Law, EXCEPT:
Response: Dowries or gifts made on account of marriage and before its celebration or within
one (1) year thereafter by parents to each of their legitimate, recognized natural or adopted
children to the extent of the first P10,000
Feedback: Correct! Under the Tax Reform Acceleration and Inclusion (TRAIN) Law, the gifts on
account of marriage have been removed.
Score: 1 out of 1 Yes
Question 6
Dana Marie, a non-resident alien decedent, died, leaving the following:

Interest in a business partnership


P250,000
organized abroad
Shares in a foreign corporation, 60% 140,000
of the business is situated in the
Philippines
Shares of a foreign corporation traded
620,000
in the Philippine Stock Exchange
Claims from citizen debtors 1,700,000
What is the total amount from the above intangible properties that shall be included in the gross
estate of Dana Marie?
Response: P1,700,000
Feedback: Incorrect! This is not the only item to be included in the gross estate of Dana Marie.
Score: 0 out of 1 No
Question 7
When Jin Garcia died, he had three (3) legal heirs: Jun Garcia, Lisa Garcia, and Doy Garcia.
Jin’s estate consisted of the following real properties to be shared equally among his legal
heirs:

Classificatio Assessor’s
Zonal Value
n Value
Residential P4,000,000 P3,000,000
Commercial 1,500,000 6,000,000
Agricultural 5,000,000 2,100,000

Instead of sharing the properties equally, the heirs executed an Extrajudicial Settlement of the
Estate. According to their agreement, the residential property went to Jun, the commercial
property went to Lisa, and finally, the agricultural property went to Doy.

How much donor’s tax will be payable, if any?


Response: P45,000
Feedback:
Correct! First, solve for the equal share of each heir:

Residential P4,000,000
Commercial 6,000,000
Agricultural 5,000,000
Total P15,000,000
Divide by: 3
An equal share of each heir P5,000,000
As can be observed, Junghwan’s share in the extrajudicial settlement is less compared to his
supposed share. Thus, he donated to his siblings:

Share in estate P5,000,000


Value received 4,000,000
Value foregone P1,000,000
Less: Exemption (250,000)
Net gift P750,000
Tax rate 6%
Donor’s tax P45,000
Score: 1 out of 1 Yes
Question 8
On February 1, 2022, Ms. Jillian Diokno donated a piece of land to her best friend, Antonio
Baguilat, whose assessed value was P1,000,000 and a zonal value of P800,000 at the time of
donation. She also donated to a charitable organization cash amounting to P200,000 and an
automobile with a purchase price of P700,000.
The piece of land was encumbered with an unpaid mortgage of P300,000, which the donee did
not assume. In addition, the donee agreed to pay the applicable donor’s tax of P210,000.
How much is the donor’s tax due?
Response: P45,000
Feedback:
Correct! Determine the value of each donation first. For the donated piece of land, the fair
market value (FMV) equals the highest of the assessed and zonal values, which is P1,000,000.
The donation to the charitable organization will be equal to their FMV at the time of donation:
P200,000 for the cash and P700,000 for the automobile.
Then, get the donor’s tax due:

Donated Land P1,000,000


Cash 200,000
Automobile 700,000
Total Gross Gift P1,900,000
Less:
Allowed deduction 250,000
Donation made to the charitable institution
900,000
(P200,000 +P700,000)
Taxable Net Gift P750,000
Multiply by: 6%
Donor’s Tax Due P45,000
Score: 1 out of 1 Yes
Question 9
Ton was 60 years old when he died intestate last month. He was survived by his 40-year-old
wife, Lisa, their 17-year-old legitimate son, Beom, and illegitimate daughter, Jennie. Moreover,
Ton had a brother named Jake, single with no children, and both of their parents are still alive.
All of the following are compulsory heirs, EXCEPT:
Response: Beom
Feedback: Incorrect! Beom, as the legitimate son of Ton, is a primary compulsory heir.
Score: 1 out of 1 Yes
Question 10
Mr. Sy died. His heirs collected the following proceeds from life insurance policies:
 Sunlife, P840,000 revocably designated to wife
 BPI AIA, P600,000, irrevocably designated to daughter
 PruLife, P500,000 revocably designated to Mr. San Juan’s estate
 AXA, P700,000, irrevocably designated to Mr. Sy’s executor

How much will proceeds from the above insurances be included in the gross estate of Mr. Sy?
Response: P2,040,000
Feedback:
Correct! The following proceeds from life insurance policies shall form part of the gross estate.

From Wells Fargo P840,000


From Maybank 500,000
From Sunlife 700,000
Total P2,040,000

VAT
Question 1
What is the form to file the monthly tax return for value-added tax (VAT)?
Response: BIR Form 2550M
Feedback: Correct! It shall be filed on the 20th day after the end of each month.
Score: 1 out of 1 Yes
Question 2
Spray Kids Inc., VAT-registered, had the following data (net of business taxes) for the last
quarter of 2021 in Philippine peso:

Quarterly
Sales October November December
Totals
Vatable Sales to private entities P500,000 P200,000 P300,000 P1,000,000
Exempt sales 400,000 200,000 600,000
Sales to the government 300,000 100,000 400,000
Export sales 200,000 50,000 250,000
Total P1,400,000 P400,000 P450,000 P2,250,000

Purchase of goods from VAT Quarterly


October November December
suppliers related to: Totals
Vatable Sales to private entities P40,000 P50,000 P90,000
Exempt sales 30,000 50,000 80,000
Sales to the government
Export sales 30,000 30,000
Total P70,000 P30,000 P100,000 P200,000

Purchase of services from VAT Quarterly


October November December
suppliers related to: Totals
Vatable Sales to private entities P20,000 P20,000
Exempt sales 30,000 30,000
Sales to the government P200,000 100,000 300,000
Export sales 20,000 20,000
Total P220,000 P50,000 P100,000 P370,000

Quarterly
October November December
Totals
Purchase of depreciable capital goods P3,000,00 P2,000,00 P5,000,00
from VAT supplier 0 0 0
Life in years 3 6
The corporation had excess input tax credit from the previous quarter in the amount of P3,900.
In November 2021, it chose to file an application for a value-added tax (VAT) refund in the
amount of P2,000. The purchase of depreciable capital goods is for the benefit of all its
business.
What is the VAT payable for the 4th Quarter of 2021?
Response: P4,276
Feedback:
Correct! This can be computed as follows:

Output VAT (P1,400,000 x 12%) P168,000


Input Tax Credit:
ITC previous quarter 3,900
Purchase of goods (120000 x 12%) 14,400
Purchase of services (340000 x 12%) 40,800
Amortized ITC allocable to VATable (38000 x
27,867
1650/2250)
Amount refunded (2,000) (84,967)
VAT due 83,033
Credits
VAT payable, Oct 17,757
Vat payable, Nov 13,000
Creditable VAT withheld (400000 x 12%) 48,000 (78,757)
VAT payable P 4,276
Score: 1 out of 1 Yes
Question 3
The following transactions have been exempted from the 12% value-added tax (VAT) under
the Corporate Recovery and Tax Incentives for Enterprises (CREATE) Act, EXCEPT:
Response: Sale of raw materials or packaging materials to export-oriented enterprises whose
export sales exceed 70% of total annual production
Feedback:
Correct! As implemented by Revenue Regulations 9-2021, 12% VAT shall apply to the
following transactions beginning June 17, 2021:
 Sales of raw materials or packaging materials to a non-resident buyer for delivery to resident
local export-oriented enterprises.
 Sales of raw materials or packaging materials to export-oriented enterprises whose export
sales exceed 70% of total annual production.
 Those considered export sales under Executive Order No. 226, the Omnibus Investment Code
of 1987, and other special laws.
 Processing, manufacturing, or repacking goods for other persons doing business outside the
Philippines, which goods are subsequently exported, where the services are paid for in
acceptable foreign currency and accounted for following the rules and regulations of the
Bangko Sentral ng Pilipinas (BSP).
 Services performed by subcontractors and/or contractors in processing, converting, or
manufacturing goods for an enterprise whose export sales exceed 70% of total annual
production.
Score: 1 out of 1 Yes
Question 4
Treasen Corp., VAT-registered, had the following data (net of business taxes) for the last
quarter of 2020 in Philippine peso:

Quarterly
Sales October November December
Totals
Vatable Sales to private entities P500,000 P200,000 P300,000 P1,000,000
Exempt sales 400,000 200,000 600,000
Sales to the government 300,000 100,000 400,000
Export sales 200,000 50,000 250,000
Total P1,400,000 P400,000 P450,000 P2,250,000

Purchase of goods from VAT Quarterly


October November December
suppliers related to: Totals
Vatable Sales to private entities P40,000 P50,000 P90,000
Exempt sales 30,000 50,000 80,000
Sales to the government
Export sales 30,000 30,000
Total P70,000 P30,000 P100,000 P200,000

Purchase of services from VAT Quarterly


October November December
suppliers related to: Totals
Vatable Sales to private entities P20,000 P20,000
Exempt sales 30,000 30,000
Sales to the government P200,000 100,000 300,000
Export sales 20,000 20,000
Total P220,000 P50,000 P100,000 P370,000

Quarterly
October November December
Totals
Purchase of depreciable capital
P3,000,000 P2,000,000 P5,000,000
goods from VAT supplier
Life in years 3 6

The corporation had excess input tax credit from the previous quarter in the amount of P3,900.
In November 2020, it chose to file an application for a value-added tax (VAT) refund in the
amount of P2,000. The purchase of depreciable capital goods is for the benefit of all its
business.

What is the VAT payable for November 2020?


Response: P13,000
Feedback:
Correct! This can be computed as follows:

Output VAT (P200,000 x 12%) P24,000

Input Tax Credit:

Purchase of goods (P30,000 x 12%) 3,600

Purchase of services (P20,000 x 12%) 2,400

Amortized ITC allocable to VATable (P14,000 x 2/4) 7,000

Amount refunded (2,000) (11,000)

VAT payable P13,000

Score: 1 out of 1 Yes


Question 5
Seven Inc., VAT-registered, had the following data (net of business taxes) for the last quarter of
2021 in Philippine peso:

Quarterly
Sales October November December
Totals
Vatable Sales to private entities P500,000 P200,000 P300,000 P1,000,000
Exempt sales 400,000 200,000 600,000
Sales to the government 300,000 100,000 400,000
Export sales 200,000 50,000 250,000
Total P1,400,000 P400,000 P450,000 P2,250,000
Purchase of goods from VAT Quarterly
October November December
suppliers related to: Totals
Vatable Sales to private entities P40,000 P50,000 P90,000
Exempt sales 30,000 50,000 80,000
Sales to the government
Export sales 30,000 30,000
Total P70,000 P30,000 P100,000 P200,000

Purchase of services from VAT Quarterly


October November December
suppliers related to: Totals
Vatable Sales to private entities P20,000 P20,000
Exempt sales 30,000 30,000
Sales to the government P200,000 100,000 300,000
Export sales 20,000 20,000
Total P220,000 P50,000 P100,000 P370,000

Quarterly
October November December
Totals
Purchase of depreciable capital
P3,000,000 P2,000,000 P5,000,000
goods from VAT supplier
Life in years 3 6

The corporation had excess input tax credit from the previous quarter in the amount of P3,900.
In November 2021, it chose to file an application for a value-added tax (VAT) refund in the
amount of P2,000. The purchase of depreciable capital goods is for the benefit of all its
business.

What is the VAT payable for October 2021?


Response: P17,757
Feedback:
Correct! This can be computed as follows:

Output VAT (P800,000 x 12%) P96,000


Input Tax Credit:
ITC previous quarter P3,900
Purchase of goods (P40,000 x 12%) 4,800
Purchase of services (P220,000 x 12%) 26,400
Amortized ITC allocable to VATable (P10,000 x 1/1.4) 7,143 (42,243)
VAT due P53,757
Creditable VAT withheld (P300,000 x 12%) (36,000)
VAT payable P17,757
Score: 1 out of 1 Yes
Question 6
iCON Corp., VAT-registered, had the following data (net of business taxes) for the last quarter
of 2020 in Philippine peso:

Quarterly
Sales October November December
Totals
Vatable Sales to private entities P500,000 P200,000 P300,000 P1,000,000
Exempt sales 400,000 200,000 600,000
Sales to the government 300,000 100,000 400,000
Export sales 200,000 50,000 250,000
Total P1,400,000 P400,000 P450,000 P2,250,000

Purchase of goods from VAT Quarterly


October November December
suppliers related to: Totals
Vatable Sales to private entities P40,000 P50,000 P90,000
Exempt sales 30,000 50,000 80,000
Sales to the government
Export sales 30,000 30,000
Total P70,000 P30,000 P100,000 P200,000

Purchase of services from VAT Quarterly


October November December
suppliers related to: Totals
Vatable Sales to private entities P20,000 P20,000
Exempt sales 30,000 30,000
Sales to the government P200,000 100,000 300,000
Export sales 20,000 20,000
Total P220,000 P50,000 P100,000 P370,000

Quarterly
October November December
Totals
Purchase of depreciable capital
P3,000,000 P2,000,000 P5,000,000
goods from VAT supplier
Life in years 3 6

The corporation had excess input tax credit from the previous quarter in the amount of P3,900.
In November 2020, it chose to file an application for a value-added tax (VAT) refund in the
amount of P2,000. The purchase of depreciable capital goods is for the benefit of all its
business.

What is the VAT payable for the 4th Quarter of 2020?


Response: P20,889
Feedback:
Correct! This can be computed as follows:

Output VAT (P1,000,000 x 12%) P120,000

Input Tax Credit:

ITC previous quarter P3,900

Purchase of goods (P120,000 x 12%) 14,400

Purchase of services (P40,000 x 12%) 4,800

Amortized ITC allocable to VATable (P38,000 x


21,111
1250/2250)

Amount refunded (2,000) (42,211)

VAT due P77,789

Credits

VAT payable, Oct P43,900

Vat payable, Nov 13,000 (56,900)

VAT payable P 20,889


Score: 1 out of 1 Yes
Question 7
Which is EXEMPTED from value-added tax (VAT)?
Response: Sale of agricultural food products in their original state
Feedback: Correct! The sale or importation of agricultural and marine food products in their
“original state,” livestock, and poultry of a kind generally used to yield or produce foods for
human consumption is exempt from value-added tax (VAT).
Score: 1 out of 1 Yes
Question 8
Engine Corp., VAT-registered, had the following data (net of business taxes) for the last quarter
of 2020 in Philippine peso:

Quarterly
Sales October November December
Totals
Vatable Sales to private entities P500,000 P200,000 P300,000 P1,000,000
Exempt sales 400,000 200,000 600,000
Sales to the government 300,000 100,000 400,000
Export sales 200,000 50,000 250,000
Total P1,400,000 P400,000 P450,000 P2,250,000

Purchase of goods from VAT Quarterly


October November December
suppliers related to: Totals
Vatable Sales to private entities P40,000 P50,000 P90,000
Exempt sales 30,000 50,000 80,000
Sales to the government
Export sales 30,000 30,000
Total P70,000 P30,000 P100,000 P200,000

Purchase of services from VAT Quarterly


October November December
suppliers related to: Totals
Vatable Sales to private entities P20,000 P20,000
Exempt sales 30,000 30,000
Sales to the government P200,000 100,000 300,000
Export sales 20,000 20,000
Total P220,000 P50,000 P100,000 P370,000
Quarterly
October November December
Totals
Purchase of depreciable capital
P3,000,000 P2,000,000 P5,000,000
goods from VAT supplier
Life in years 3 6

The corporation had excess input tax credit from the previous quarter in the amount of P3,900.
In November 2020, it chose to file an application for a value-added tax (VAT) refund in the
amount of P2,000. The purchase of depreciable capital goods is for the benefit of all its
business.

What is the VAT payable for October 2020?


Response: P43,900
Feedback:
Correct! This can be computed as follows:

Output VAT (P500,000 x 12%) P60,000


Input Tax Credit (ITC):
ITC previous quarter 3,900
Purchase of goods (P40,000 x 12%) 4,800
Purchase of services (P20,000 x 12%) 2,400
Amortized ITC allocable to VATable (P10,000 x
5,000 (16,100)
7/14)
VAT payable P43,900
Score: 1 out of 1 Yes
Question 9
Which is NOT a transaction deemed sale?
Response: Consignment of goods if actual sale is made within 60 days following the date such
goods were consigned
Feedback: Correct! If the consigned goods are sold within 60 days, the transaction is not
considered a deemed sale but a direct consumption in which the seller should report the
amount as part of its gross sales.
Score: 1 out of 1 Yes
Question 10
Teasen Corporation has the following sales during the month:
Sale to private entities P224,000
Sale to export-oriented enterprises 100,000
Sale of exempt goods 100,000

The following input taxes were passed on by its value-added tax (VAT) suppliers during the
month:

Input tax on taxable goods P5,000


Input tax on sale of export-oriented enterprises 3,000
Input tax on sle of exempt goods 2,000
Input tax on depreciable capital good not attributable to any specific
20,000
activity

The VAT payable for the month is:


Response: P1,000
Feedback:
Correct! First, compute the output value-added tax (VAT).

Sale to private entities (P224,000 x12/112) P24,000


Sale to export-oriented enterprises (P100,000 x
0
0%)
Total Output VAT P24,000

Then, determine the total creditable input VAT.

Sale to private entities


P5,000

Sale to export-oriented enterprises 3,000


Input tax on depreciable capital good not
attributable to any vatable transactions 15,000
(P20,000 x 300,000/400,0000)
Total Input VAT P23,000

VAT payable:

Total Output VAT P24,000


Less: Total Input VAT 23,000
VAT Payable P1,000
Score: 1 out of 1

PERCENTAGE AND LOCAL TAXES


Question 1
All of the following are examples of the scope of Local Taxation, EXCEPT:
Response: Imposition of P20 basic community tax on all individuals in the Municipality of
Manila
Feedback: Correct! The basic community tax for individuals must be five pesos (P5.00) only.
Score: 1 out of 1 Yes
Question 2
Mr. Panay, single and a resident of Loay, Bohol, had the following data from the preceding
taxable year:

Income from businesses located in the


P4,500,000
Philippines
The assessed value of business properties in
2,500,000
the Philippines
Income from a business in India 3,000,000
The assessed value of business properties in
2,000,000
India
Compensation Income from Holy Name
200,000
University (HNU)

How much is Mr. Panay’s total community tax?


Response: P4,705
Feedback:
Correct! Only those incomes within the Philippines will be subjected to the community tax.
Thus:

Basic Community Tax P5


Additional Community Tax
Income from Business in the Philippines
4,500
(P4,500,000/1,000) x 1
Compensation income
200
(P200,000/1000) x1
Total Community Tax P4,705
Score: 1 out of 1 Yes
Question 3
Which of the following is the additional community tax on corporations?
Response: Two pesos (P2.00) for every P5,000
Feedback: Correct! Two pesos (P2.00) for every P5,000 of a) the assessed value of real
property in the Philippines owned by a corporation; b) gross receipts or earnings from business
in the Philippines by a corporation.
Score: 1 out of 1 Yes
Question 4
Which of the following is subject to common carrier’s tax?
Response:
Common carriers by land for the transport of passengers
Feedback: Correct! If the common carriers by land are for the transportation of passengers,
they will be subjected to percentage tax under Section 117 of the Tax Code. On the other hand,
if the common carriers by land are for cargo transport, they will be subjected to value-added tax
(VAT).
Score: 1 out of 1 Yes
Question 5
Which of the following is NOT subject to the 3% percentage tax?
Response: Franchise grantees of electric utilities
Feedback: Correct! They will be subject to the 5% franchise tax.
Score: 1 out of 1 Yes
Question 6
All of the following are required to file the Bureau of Internal Revenue (BIR) FORM 2000 for
their one-time transactions, EXCEPT:
Response: Regularly employed on a wage or salary basis for at least 30 consecutive working
days during any calendar year
Feedback: Correct! This requirement is for the community tax. They will be required to pay P5
basic community tax.
Score: 1 out of 1 Yes
Question 7
The amount of percentage tax to be imposed on franchise grantees of radio and television
broadcasting companies whose annual gross receipts exceed P10,000,000 is:
Response: Exempt
Feedback: Correct! The said franchise grantees of radio and television will be subjected to
value-added tax (VAT) since their annual gross income already exceeds the P10,000,000
ceiling.
Score: 1 out of 1 Yes
Question 8
Mariana operates a cockpit. Inside the cockpit, she also runs a restaurant. Data for the
particular quarter is as follows:

Gross Receipts from:


Cockpit operations P500,000
Sale of foods in restaurant operations P100,000
Sale of liquor in the restaurant
P150,000
operations

The amusement tax due of Mariana is:


Response: P90,000
Feedback: Incorrect! It is not the only gross receipt from cockpit operations that will be
subjected to the amusement tax.
Score: 0 out of 1 No
Question 9
Which of the following governs the taxing power and other revenue-raising powers exercised
by provinces, cities, municipalities, and barangays?
Response: Local Government
Feedback: Correct! The objective of delegating taxation power to local governments is to
promote the local government unit's self-sufficiency and strengthen and make local
government units more autonomous.
Score: 1 out of 1 Yes
Question 10
It is the amusement tax to be imposed on boxing exhibitions.
Response: 18%
Feedback: Incorrect! This is the percentage of amusement tax for cockpits, cabarets, and
day/night clubs.
Score: 0 out of 1 No
Assignment
 Type: Quiz
 Max score: 10
Score

Your best submission is used

1.75
Category Progress
Grade 80
None 20
8/10 (80%)

Congratulations, you have completed this assignment.

You might also like